You are on page 1of 141

‫ال اله اال انت سبحانك اني كنت من الظاملني‬

1- Which of the following have means that individuals have the capacity to
understand basic health information to make appropriate health decisions?
Health literacy
‫الس'م عليكم‬
‫كل عام وانتم بخير‬
2- Methotrexate antidote:
‫حمد العضيله‬
Leucovorin

3- Case about beta-blocker toxicity and asking about antidote:


Glucagon

4- What is the antidote for heparin toxicity?


Protamine sulfate

5- Case about toxicity and the doctor give the patient Deferoxamine, what is the
toxicity:
Iron toxicity

https://t.me/splestudyplan
6- Antidote of paracetamol:
N-Acetyl cysteine (NAC)

7- Which of the following is consider tertiary resource:


Review article

8- Case about drug cause adverse effect, which of the following resource would be
appropriate for determine side effect:
drugs

Safi ah a IET W

Al ghamdi
9- Case about female patient with hypertension on lisinopril, she become
pregnant:
Switch to methyldopa

10- Case about female patient with DVT on warfarin, she become pregnant:
Switch to enoxaparin

11- Patient with malaria, what is the treatment: https://t.me/SLPESTUDY


Chloroquine

12- Which of the following laxative is contraindicated during pregnancy:


Castor oil

13- Following the ethical principle of nonmaleficence requires that


Pharmacists:
do good to patients, placing the benefit of the patient above all else

14- Case about anaphylaxis, what type of hypersensitivity:


Immediate hypersensitivity

15- IgG responsible in which type of hypersensitivity:


Type 2

16- Which of the following is the first antibody to be produced new infection:
IgM

17- Which of the following white blood cell types is the target of immunodeficiency
virus during acquired immune deficiency syndrome(AIDS)?
CD4 lymphocytes
18-
Physiological < Safety < Love and belonging < Esteem < Self actualization

off
19- Coagulase use to differentiate between:
on a
S.aureus and S.epidermidis G Ul

20- Which of the following is Gram negative bacilli:


E.coli
a Pseudomonas

21- Cost of hospitalization:


Direct medical

22- Humanistic outcome, what is the outcome measurement unit:


Quality adjusted life year

23- Case about patient HLA-B (1502) positive cause Steven-Johnson-syndrome,


which drug is responsible:
Carbamazepine

24- Which of the following is a common enzyme polymorphism that could


potentially alter clopidogrel efficacy?
CYP2C19

25- High alert medication:


Insulin

26- Which of the following does not require sterilization:


Oral tablet
27- :
Promotion

28- Anti-psychotic cause agranulocytosis:


Clozapine

29- Patient on warfarin and INR still pending, what can you monitor:
Gum bleeding

30- Which of the following vaccines is contraindicated in pregnancy:


MMR

31- IPV can not be given to patient who has allergy to:
Neomycin

32- Which of the following vaccine is strongly recommended before Haj:


Meningococcal vaccine

33- Vitamin A:
Retinol

34- Vitamin D active form:


1,25-dihydrocycholecalciferol

35- USP 797 used for:


Sterile preparation
36- USP 795 used for:
Non-Sterile preparation

37- Which of the following measurements is used to assess patient adherence:


Medication event system

38- Which of the following is the type of studies considers data from multiple
studies of different designs:
Systematic review

39- Which of the following medication error is difficult to prevent?


Allergic reactions

40- The authors report two isolated gastro-intestinal tuberculosis in renal


transplant recipients that illustrates the difficulty of making this diagnosis and a
brief review of the literature on its clinical presentation, diagnosis and
therapeutic approach. What is the type of study in the above-mentioned
abstract?
Case-series

41- Which of the following terms describes living microbe with reduced virulence
,and used for vaccination?
Attenuated

42- which of the following systems can be integrated with computerized physician
order entry to guide physician during prescribing medications?
Clinical decision support system
43- Which of the following method is successful in reducing medication error and
enhance patient safety in hospitals?
Implementation of computerized physician order entry (CPOE) system

44- Which of the following is a pharmacological property of chlorpromazine?


anti-emetic drug

45- Which of the following lab results support a diagnosis of primary


hypothyroidism:
elevated TSH and low free T4

46- A 7-year-old girl is brought to the clinic with complain of itchy scalp.
Examination shows lice infestation. What is the best management?
dimethicone lotion

47- Which of the following cannot be used safely in a patient with sulfa allergy:
Gliclazide

48- Which of the following describe glucose uptake and usage by as compared to
other tissues of the body?
Brain cells can uptake and use glucose without being stimulated by insulin

49- type of immunity passed from mother to baby:


Passive natural immunity

50- case in pharmacy requires private area and the pharmacy do not have private
area, what is the type of barrier:
Environmental barrier
51- which of the following is Hypotonic solution:
0.45% Sodium chloride

52- A 28-year-old woman was recently diagnosed with Crohn was started with
high dose intervenors corticosteroids .she conversion to oral drugs and was
started on infliximab, a tumor factor (TNf) inhibitor. Which of the following
tests should be performed prior to infliximab?
Tuberculosis screening

53- Population of 10,000 was screen for disease 500 have the disease:
500/10,000*100 = 5%

54- the appropriate time to Withdraw a trough vancomycin level for a patient:
before 4th dose

55- which of the following insulins has the longest duration of action:
Glargine

56- Which of the following is the recommended anti-coagulation strategy for atrial
fibrillation patient with a CHA2DS2-VACS score of 5?
Dabigatran

57- Which of the following is most appropriate to give hepatitis B vaccine for
unvaccinated patient?
3 doses vaccine series administered intramuscularly at 0, 1, and 6 months

58- Which of the following medications to be used under caution is in patients with
ischemic heart disease?
Celecoxib
59- A 60-year-old man with 6-year history of myocardial infarction and gout. He
presented to the clinic complain of severe pain in his first left consistent with
acute gout flare. He mentions that he can't colchicine. What is the most
appropriate pharmacotherapy recommendation for his acute flare-associated
pain? correct answer:: Acetaminophen
Ibuprofen
paracetamol
op
60- Which of the following medications for osteoporosis will significantly reduce
the risk of hip fracture?
Risedronate

61- A 36-year-old man not known to have any medical illness diagnosed recently
with an active pulmonary tuberculosis. Which of the following is the best
regimen to start?
Isoniazid, rifampin, ethambutol and pyrazinamide

62- Canagliflozin MOA:


Sodium /glucose co-transporter 2 inhibitor
SOLT Z i

63- Amlodipine side effect:


Peripheral edema

64- Which of the following cause hyperkalemia:


Spironolactone

65- Which of the following cannot give in combination:


Lisinopril and losartan
AR B t A CEI
66- Which of the following cannot be used in patient with eGFR 20:
Spironolactone
67- Which of the following is a potential side effect of chlorothiazide?
Hypokalemia

68- Which of the following herbal products can potentiate the effects of oral
antidiabetic?
Ginseng

his 0TH
69- Which of the following is the pharmacological property of atenolol?
cardio selective beta 1-blocker

70- Which of the following medications is an estrogen inhibitor used for the
treatment of breast cancer:
Tamoxifen

71- Itraconazole capsules require acidic pH optimal dissolution and absorption.


Which of the following is an appropriate advice to optimize the oral absorption
of the drug?
take it with food

72- A 12-year-old boy is brought to the Emergency with complaints of inability to


open eyes and difficulty in breathing. History revealed snake bite. Examination
shows anxiouus, tachycardia and tachypnea, bilateral ptosis, no bite mark seen
and no swelling of the.limb (see lab result).
give Anti-snake venom (ASV) plus Neostigmine and keep under observation

73- Which of the following medications produces antiplatelet effect by inhibiting


Adenosine Diphosphate (ADP) receptors?
Clopidogrel
74- To ensure safe and proper disposal of hazardous waste, a color- coded system
was created. Which of the following colors is the color of hazardous waste
containers?
Yellow

75- Which of the following is recommended regarding air bubble in the syringe for
a patient starting on enoxaparin subcutaneous Injection?
inject the dose with the air bubble

76- An extemporaneous drug label says to store the drug between 15 and 25
degrees centigrade. Which of the following is the most appropriate place
where this drug should be stored? Refrigerator 8C – 15C
8C –15C cool room correct answer Room temp.
Freeze. -10C – -25C

77- Acetaminophen is an antipyretic and an analgesic drug. It binds to a cytosolic


protein (see image). Which of the following types of bonds is formed between
the protonated amine of the acetaminophen and the cytosolic acid of the
aspartate residue of the protein?
hydrogen bond

78- The ionization of the drug is very important to understand the


pharmacodynamic of the drug. Which of the following forms are absorbed from
the stomach?
non polar-unionized

79- Which of the following is the exact date for a drug to be safety taken if its
expiration date 8/2018:
31 August 2018

80- Maximum dose of paracetamol in adult:


4g
81- Maximum dose of ibuprofen OTC in adult:
1200mg

82- Child with otitis media, what is treatment:


High dose amoxicillin

83- Case about child with otitis media, what the right regimen:
High dose amoxicillin for 7 days

84- Which of the following is considered intentional nonadherence:


Afraid of side effects

85- Which of the following can be used as mood stabilizer in bipolar:


Lamotrigine

86- simvastatin when to take:


At bedtime

87- 1 g dextrose kcal:


3.4

88- Mid goal to Ion term future goal:


Vision

89- Which of the following can be used for UTI:


Cranberry
90- case about one using two drugs of cancer one with less side effect to choose
what is cost:
cost-effectiveness (not sure)

91- patient on warfarin and has mechanical valve and INR 3.5 what should you do:
no treatment

92- which of the following is important for safety of drug:


trough level

93- Your colleagues have been contacted with c.difficile patient and used
disinfectant wipes what to do:
A Wash hand with alcohol and water (not sure) SOAP

94- An 8-year-old child had difficulty in breathing. swelling of lips and tongue,
which type of hypersensitivity:
Immediate hypersensitivity

95- pharmacist wrote email to other pharmacist to ask add medication to the
formulary in the hospital this is consider as:
inter-institutional
anecommunication (not sure) wVttieu Communication
96- COPD patient category B, what is the treatment:
Tiotropium

97- COPD score:


mMRC

98- Which contraceptives have high dose of estrogen and contraindication in DVT
women:
Transdermal patch
99- Head of pharmacy decide to make group to arrange formulary drugs in hospital:
Planning (not sure)

100- Case about invasive fungal infection aspergillosis what is the treatment:
Voriconazole

101- clostridium difficile cause:


pseudomembranous colitis

102- Which the fallowing treatment thrombocytopenia:


Argatroban

103- function:
Help in produce protein

104- Human cell membrane consists of:


Phospholipids and cholesterol

105- Case of osteomyelitis, what is the duration of treatment:


4-6 weeks

106- Lithium required monitoring of:


eGFR, calcium and Thyroid function.

107- Case about patient with multiple conditions and taking a lot of drugs one
of them cause problem:
Adverse drug reaction
108- Calculate Crcl of male patient:
X wt
140 age
30 ml/min

109-
EE
40 years old female, her weight 80, Scr = 0.5, Calculate Crcl

XO 85
188.8
i a
110- Heparin, what to monitor: https://t.me/SLPESTUDY
APTT activated partial thromboplastin time

111- Which of the following is the side effect of using fluticasone inhaler:
Oral candidiasis

112- Consulting point when using fluticasone inhaler:


Wash your mouth with water after each use

113- Which of the following describe tolerance:


increasing dose of drug to obtain the same effect

114- A patient purchasing sublingual nitroglycerin tablets should be told to


store the medication:
in an amber glass bottle with a metal cap

115- enzyme kinetics law:


Michaelis-Menten

116- Case about patient with multiple condition on lisinopril, gentamicin and
furosemide, the interaction between which drug:
Gentamicin and furosemide
117- Using aspirin in child:

118- Case about patient with allergy and she has exam, what is the best
treatment:
Second generation anti-histamine

119- Which of the following is the best way to avoid medication error:
Electronic prescription

120- Which study design is used for identifying causality:


RCT

121- Case about researcher want to evaluate risk factor:

RF
Prospective cohort study
co hort pro
122- Case about patient taking simvastatin and it cause myopathy, what is the
test:
Creatin kinase

123- Which of the following can benefit patient with DM:


Ginseng

124- Case about patient with palpitation, what is the treatment:


Anti-arrhythmic

125- Structure of new drug for Alzheimer disease, what is your


recommendation, can we enter the drug in the formulary:
I do not know the answer

126- Two Structures, what is the reaction:


Oxidation
127- Which of the following structure is the active form of vitamin D:

I 25 di OH Cholcalcifer

OH

Calculation:
- 2 questions about Vd (C and amount of drug is given in the question)
- Tmax
- BMI
- Crcl
- Cl
- Number of tablets for prescription
- 6 questions give you g or ml of drug (example: suspension), calculate how many
ml or g required for patient (focus on the unit)
- Absolute bioavailability
Jordan OMSB
Date : 31-3-2022
Function of Co-enzyme Q converts the energy in carbohydrates and fatty
acids into ATP to drive cellular machinery and synthesis

Resource Planning definition


Pharmacy manager gather employees to set plan how to sell certain
product ? This is called : Planning - Organizing
Cockcroft-Gault equation CrCl (male) = ([140-age] × weight in kg)/(serum
creatinine × 72)

Error done intentionally by patient ? Omission-Forgetfulness


Non-adherence example
Vaccine for minor injury
Live attenuated vaccine example ??
measles, mumps, rubella (MMR),yellow fever, influenza intranasal vaccine), and
oral polio vaccine

Vaccine given to all ages


Vaccine contraindicated in pregnancy??
HPV
MMR
Varicella
ZVL
BCG
LAIV

Itchy scalp with lice ? Primethrin - Dimethicone lotion


Atrial Flutter drug of choice
DVT medication ? Enoxaparin
Phenytoin side effect ? Gingival Hyperplasia
Patient taking multiple medications for hypertension, depression and

effect ? Serotonin Syndrome


Anti Arrhythmic causing lupus syndrome Quinidine and Procainamide
Hashimoto disease cause ? Tachycardia
bradycardia
A case including side effect of ACE inhibitors, determine which drug
HIT treatment argatroban
Aspirin given for which case ? Child 4 years , adult above 55 , Renal
patient and one more I forgot
half lives eliminates ? 97% of drug
To obtain therapeutic effect pharmacy should activate the system of
communication between therapist and prescriber
Anticoagulant for Diabetic patient
Aspirin structure hydrogen bond
Drug causes c.difficle ? Clindamycin , Vancomycin was on choices
Group-B streptococcus treatment but penicillin sensitive
Inhaled Anthrax ? Ciprofloxacin
MOA of azoles
Bacteria in nails
Added to yeast ? Forgot the options
Acetaminophen antidote acetylcysteine
Descriptive study type
Cross sectional study , prospective cohort study examples ? Mention
the type of study
A case of high BP and normal potassium level, what to do
Panadol maximum dose 4 grams (4000 mg) per day.
calculations : 2 on t1/2 , 2 on infusion rate , 1 on Clearance
Child coming with Amoxicillin prescription, what is the dose for Otitis
Media ? How many bottles
Category B definition
Multiple antibiotics Category B,C,D ? Which drug is safe in pregnancy
Supplement given with corticosteroids in Osteoporosis Calcium
and vitamin D

Company gave amount of money to employee because he achieved his


target ? Reward
Highest intensity PPI
Meyler s side effects of drugs
Gram+ve example ? Actinomycete
H1N1 ? Oseltamivir
HLB-A
High alert medication definition drugs that bear a heightened risk of
causing significant patient harm when they are used in error

How to reduce errors in LASA medications Minimize the LASA drug


selections from the selection screen and flag them as LASA
medications.

Period between ACE inhibitors & start Entresto ? 36 hours


in 1 dose means ? Dextrose & Amino Acids
Generate impulses ( pacemaker ) ? SA node
dextrose is high alert solution
T-Lymphocytes maturation ? Bone Marrow
Amphetamines MOA increasing the amounts of dopamine,
norepinephrine, and serotonin (to a lesser extent) in the synaptic cleft
through a variety of mechanisms

Lithium side effect Gl upset (nausea, abdominal pain), cognitive


effects, cogwheel rigidity
tremor, weight gain, polyuria/polydipsia, hypothyroidism

P value indicates significance


QT indicates ventricular contraction and relaxation
To conduct study we need ? healthy volunteers
Management objectives
Confounding
Prescription is valid to dispense for 72 hours Oral
Phenazine class
Drug helps you quit smoking
Citalopram side effect
Somnolence, insomnia, nausea, dry mouth, diaphoresis (dose-related),
weakness,
tremor, dizziness, headache

Which is more safe Atenolol or Propranolol


A case at which one side effect is Tachycardia ? CCB or B-blockers
A case of heart failure ? What to do ? Increase Digoxin or 3 other
options I forgot
Class II anti arrhythmic drug example Beta-blockers
Iso sorbide di nitrate side effect
Dipyridamole MOA inhibition of platelet cAMP-phosphodiesterase..
Glimepiride MOA stimulating the release of insulin from functioning
pancreatic beta cells.

A case of patient taking Metformin with high serum creatinine


MRSA treatment vancomycin
Hospital acquired infection name of bacteria
staph aureus pseudowas
Acute otitis media DOC ? High dose of Amoxicillin
Antibiotic needs adjustment ? Imipenem
A case about Vancomycin infusion rate
Human papilloma virus used in cervical cancer
Potassium Iodide is added to treat Hyperthyroidism
Purine bases ( A - G ) adenine and guanine,
Patient on osteoarthritis first line therapy Acetaminophen
Septic shock treatment Fluid
Tape worm treatment Albendazole
Methotrexate MOA in oncology inhibition of enzymes responsible for
nucleotide synthesis

- Velocity of Reaction
Michaelis-Menten equation
- Thalidomide Kefauver
- Aminotic Fluid normal range
- Cipeofloxacin SAR
Harris
5 25cm amendment
- Diuretics SAR
- Longest Duration BZN
- Norepinephrin Structure metabolism
- Cyclosproin,Vancomycin,Gentamicin Trough
- Melanoyl Co A vit ..... Biotin
- Study Design Cases
- Social Questions ....
-Emulsion ...Acasia
-Leucin .. Anorexia
-Andexate alpha for direct xa inhibitor
- Test for diaylsis patient (serum ferritin , TSAT,Hgb)
- St John wart effect with Warfarin
- You should not be given infliximab if you have active TB
- Gloves ... Isopropyl 70%
70
popyl
- Hood .... Ethanol 70%
- Inspectiom .. Palpitation .. Percussion .. Auscultation
- MOA doxorubicin inhibits topoisomerase II
-humans, chromosomes 13, 14, 15, 21, and 22 what is .... . acrocentric (
centromere is l quite near one end)
-needs a double check ....Anti thrombosis
- side effect of dapagliflozin ..Ketoacidosis
- Rivaroxaban .. take with food
- Heparin aPTT

I
- Enoxaparin Anti Factor XI
- Bivalirudin aPTT
- Warfarin INR
- BALJETS TEST to identify the cardio
digitalis
-Bentonite .... Suspending agent
- Shelf life definition the length of time a product may be stored without
becoming unsuitable for use or consumption

-Expire date= chemical stability


- BUD=both chemical and microbial stability
- Vaccine put in frezeer
bogged
- -2°C to -8°C
date - Brisk Walking ... Moderate Activity
-What to consider before calculating TPN? Propofol
-Anti-Xa levels may be recommended in underweight, obese, pregnant,
or renally impaired patients.
-Polypharmacy (PP): the use of five to nine drugs.

-initial Regimon for NTEMI? Ispirin 325+ ticagrelorl 90 g bid


- Pharmacovigilance at which phase of clinical trials!? Phase 4

-Epinephrine? Hormone and neurotransmitter


-warfarin effect with garlic, ginger, ginkgo, St. John's wort, and ginseng
-Ipeca = Ambebiasis
-Lubricant (Mg stearate ) increase flow of powder during manufacturing.
-pin worm ... mebendazole
-INR target
- Anaesthic with hepatic tox ... isofluran + halothan
- Minoxidil ... Dilate Artery Only
- Alkaloid test ... Dragondroff's test
- Septic Shock .. Fluid>NE>Dopamin
-Simvastain increase risk of rhabdomyolsis with SLCOB
-RSV prophylaxis .... Palivizumab
Digitalis Test name
Baljet
Genomics .....Abacavir HLA b 5701 , Clopidogrel Cyp2c19 , Warfarin
Vorck1 ,

Structers almost 7 question in first section:


Imipramine
Asprin acetyl group SAR
Anthestamine HTN
atenolol
Celecoxib

Calculation:

& the other one women physican prescribe for her meropinem and ask
for dose adjusment according to Crcl
Ask for T1/2 and vd and cl was given
Meq
Ask about how many ml adminsterd for all day prescription (conc of
ampoule was given)
Time for drug to be 85% cleared from the body
Patient on 40 mg furosemide and the doctor want to discharge him on
oral what is the dose?
80 9 Iv
Calculate TI from dose response curve P 2li
Absolute Bioavaliabilty (the dose for oral same as for IV)
Other calculation was direct and easy (only be careful for units)

Limitation of CPOE system :


System downtime
Not reduce medication error

Type of errore exclusive done by pharmacist:

O
Dispensing error
Which type of contraceptive have the highest amount estrogen:
Batch
7 patch
IUD
Which on is narcotic :
Fentnyl
Lorazepam
Diazepam
control
USP Sterile preparation :
USP 797
which one inhibit gluconeogenesis:
Insulin
Patient had constipation:
Increase protien intake
Increase water intake
Decrease fibers
validity of narcotic Rx:
7day
G protein coupled receptor second messenger
IP3
cGMP
cAMP

Which one associated with first dose syncope:


Terazosin
patient has cough or chest pain which plants will benefit
eucalyptus oil
Kava

phenytoin theraputic range :

I
10-20 mcg/mL
10 20

Traveler have jet lag what do you recommend?


Zolpedim
Melatonin

the drug recommended to be store in -8 dgree should be store in :


In freezer
refrigerator
Freeze drying

a
Vacine contraindicated in prenant :

pregnant
Polio
Varcella

Vitamin B name :
Cyanocobalamin B 12
stomatitis occur due to deficiency of:
Vitamin D
BZ
Vitamin B Z
(Especially Riboflavin deficiency)
T
BZ
A1C 11 what is the diagnosis?
T2DM

Canagliflozin heart effect from forest plot *

patient with +ve mastasis which empiric antibiotic is appropriate :


Cephalexin
encaupsulated bacteria :
Klebsiella

Antibiotic darken the urine :


Nitrofurantoin and Metronidazole
-which one is contraindicated in pregnant :
Statin
budget definition & other question for strategic plan definition
Which Immunoglobulin can across placenta :
IgG
which cell mediate hypersensitivity reaction :
Mast cell (Type 1 )
Natural supplement for pregnant with morning sickness :
Ginger capsule
which one contraindicated in sever sulfa allergy : furosemide

Case about patient have active TB :


rifampin , isoniazid , pyrazynamide,ethambutol
which one of NSAID have least GI side effect:

o
Celecoxib

What is the most practical method for sterilizing ophtalmic solution :


membrane filtraion through 0.2 um filter

absulte refractory period : ARTS QT


absolute refractory period QRS complex

patient started on anthracycline and cyclophosphamide what to monitor


?
ECG
Ovary perservation

INR pending what to ask the patient:


Gum bleeding

cyclophosphamide toxicity:
Hemorrhagic cystitis

type l in pharmacutics represent :


High solubility low permeability
Low solubility high permeability
Low solubility low permeability
High solubility high permeability

2 question about cost benefit analysis (dierct and easy)


case ask about the dose & frequncy of cefotriaxone in treatment of
maningitis

2 question aboute Antidote of


(organophosphate
Atropine followed by Pralidoxime (2-PAM)
, benzatrope)

Patient man wt= 80kg have DVT on warfarin what is the dose of
enoxaparin:
Enoxaparin 80 mg BID
How Ebola transmeted:
Blood & body secretions

patient sta
excepcted effect on INR:
Decrease INR

hormone responsible of milk production :


Prolactin

Contraindication of ACEI:
History of angiodema
Aortic stenosis

Flashtab technology is used to :


Fast relase tablet
Patient have RA not taking Hydroxyqluroqine tablets what is the DRPs in
the case:
Nonadherance

Patient came to hospital and you as a pharmacist find his BP 175/ 86


what you will do?
-tell him to see HCP within the week
-ask the nurse to triage and book for him an urgent appointment with
his physician

size of prostate
patient have BPH score 16 what to give:
Tamsulosin
Finasteride
Finasteride And tamsulosin
Amebia choice :
Ipeacac

patient have breast cancer HER+ what to give:


Trastuzumab
Tamoxifen

patient have minor wound and not mention the history what to give at
this time:
Tetanus vaccine only
Td
Tetanous vaccine +TIGB
Case about patient have sever chest pain while see the TV , on ECG ST
segment elevated and tropinin was high what is the diagnosis?

STEMI
ST elevation myocardial infraction

Buspirone MOA

myalgia is side effect of :


Statin

-Established COPD patient, uses O2 at home, comes in with shortness of


breath, high sCr, high RR, what does this patient have? + low
albumin
COPD exacerbation e renal failure

angery patient in community pharmacy line what to do ?


tell the other patient to call the police
Agree with patient on the truth without justification

Women came to community pharmacy and said you sell the cosmetics in
higher price than the others and pharmacist said yes
we are selling them in higher price because they are original, what type
of answer?
Aggressive
Assertive
Confirmatory to cheat
Passive
Particle need more than 10000 part of solvent?

o
Insoluble
Phase 2 occur in?

Cytosol

Produce energy?

Mitochondria
Ethambutol cause?

Loss of green red colour vision

For RA?

MTX 7.5mg weekly with folic acid 5mg daily except day of mtx

Iron deficiency anaemia cause ?

A. Angular stomatitis

Doxorubicin mechanism?
A. Alkylating
B. Antimetabolite
C. Metallating
D. ANTHRACYCLINES

A. Insulin
B. Heparin
C. Amoxicillin
D glycopyrronate

Used with INH?


B6

Dose of calcium in pregnant? 1200


sterile
What usp chapter for compounding?
USP 797

Mast cell activating receptors


A. Beta 2 receptors
B. Cytokines

Type of cell involved in type 1 ?

Mast cell

Type of cell involved in type 4 hypersensitivity?


T cell

Children with osteoporosis and dyslipidemia take?


Niacin and calcium or vit d

Vit contraindicated with high doses in pregnancy?

D
Vit A

inhibitor
cyp
Effect of cimetidine on a drug ?

Decrease metabolism

Insulin secreted by pancreas and targeted many different cells


A. Intra
B. Endocrine signaling

Tetracycline degradation at?

Acidic ph
?
Increase lipophilicity

How to decrease duration of BDZ

Add OH gp on C3

Least dominant tautemer of Histamine

Phenylephrine mechanism of action


1-adrenoceptors in the arterioles of the nasal
mucosa.

oh

Hydrogen bond

major active metabolite of diclofenac

oseltamivir
esterase
cooh

Pt own medication ?
A. Medication brought to hospital by pt or any relatives
B. Medication from
8-15 c is?
A cool place
B. Freezing
C. Refrigerating
D. Warm

Can be stored in freezing unit


Varicella vaccine

Ttt of amebiasis in pregnant , in first trimester ?


A. Tinidazol 2 gm once for 5 days
B. Notazoxanide 500mg BID for 10days
C. Hydroxy chloroquine 500mg for 7 days
D. Paromomycin 30mg/kg

Prevent of Hep A
Wash hand

Decrease risk of cervical cancer


Human papilloma virus vaccine HPV

e
Rota contraindicated in Latex allergy

colistin
kidney

oral?? rota

:
Topical NSAID

Topicam antibiotic
Tr
Breast Cancer female with HLA-B over expression so use ?
A. Tamoxifen
B. Trastuzumab 2
Status asthmaticus is an sever inflammatory case so use ?
C
A. Methylprednisolone
B. Salmeterol
C. Theophylline

M neuropathic
Pt with seizures and DM and has neck or back pain so use ? pain
A. Bupropion

0
B. Duloxetine
C. Amitriptyline
SNR l

Entacapone mechanism?
i
A. Inhibit catechol -o-methyl transferase
COMT
MTX monitoring ?
Liver function test every 6 months
LET

Prophylaxis from migraine aura?


A. Ergotamine 4 mg
B. Sumatriptan 50mg BID 7 Ttt
O
C. Propranolol 20mg BID
r
Function of carbidopa?

OA. Inhibit dopamine decarboxylase (


B. Inhibit MAO
)

C. Inhibit. COMT
Pt with schizophrenia and take drug act as dopamine antagonist
so he take ?
A. Clozapine

OB. Haloperidol
C. Another different drug

Pt take sulfamethoxazole and develop allergy so not take ?


Captopril

Pt developed arthritis problem after taking antibiotic(something


like that) , what he was taken?
A. Amoxicillin
B. Ciprofloxacin
C. Another antibiotic

Child 2 yrs with penicillin allergy so use ?


A. Tetracycline
B. Azithromycin
C. Ciprofloxacin

Drug x acting on 30s and inhibit....


So it inhibit?
Initiation

Pt with fever , shills , another symptoms , brought to hospital and


physician started empirical therapy ceftriaxone and linezolid ,
after culture was taken , mecithillin sensitivite staph so?
A. Continue and add antifungal
B. Discontinue and add meropenem
C. Discontinue and add pipracillin
long1kg 120 ng
Child 1 yr , 12 kg , has fever , physician prescribed paracetamol ,
we have strength of 250mg/5ml so he should take ?

OA. 2.5ml
B. 7.5
z go ng s l
C. 10
D. 15
9 120mg X

off 2.4 I
3 yrs old child with otitis media , physician prescribed amoxicillin ,
duration is ?
A. 10 days
B. 7 days
C. 5 days

Aerotolerant bacteria means? , can live with o2


an organism that does not require oxygen for growth but can
tolerate its presence

Lyme vector? flea


Anthrax transmission, cattle

I
Fungal cell wall chitin or ergosterol?

MMT moA, peroxidase inhibitor

Why finsteride should be d/c when want to conceive?


-risk of hypotension
-abortion causing
- affect uterine vessels

O
-teratogenic

Longest Half life ig? IGg

Tigonellin present in anise or fuingreek?

I C utero
Who convert tryposngen to trypsin? HCL
peptidase
Secrtin role bicarbonate release
The master budget is a comprehensive financial planning
document. called?
-sales budget
- operation budget
h
Narnjo scale 3 means? probable possible
2nd compartment? adipose
f

USP means by alcohol:


-Ethyl 75%

schwartz equation describe: - crcl in chidren

avoid
Levodpa counseling? embty stomach
high prolei
Levothyroxine when to monitor t4 after starting? after
1week
4 weeks
Ferrus counseling, with orange juice

Glucoma open? Pilocarp ice


Closed? Acetazolamide

Treatment of diabetes insipidus?


A. Desmopressin

Why we do Allopurinol genetic test (HLA-B*5801), to prevent


which reaction?
A. Cutaneous

O
Treatment of amoeba ? A. Ipecac
Which one is mast cell inhibitory receptor?

O
A. Siglec-8 is an inhibitory receptor expressed on mast cell

Which one is mast cell activator receptor?

O
A. Immunoglobulin independent is mast cell activation

What part of an immunoglobulin is responsible for recognizing


foreign things?

Variable FC

o Constrict capillaries
o Stimulate gastric secretions

Timeed solid

Case patient with diarrhoea, what is the reason ?


A. Mg
FlashTab technology is used in making tablets means?

Fast release

Anticancer for solid tumor?

Entrectinib,Pembrolizumab

Dose of alendronate :
( prophylaxis 35mg Po weekly

(Treatment 70mg Po weekly


Which of the following immunoglobulin (Ig) is frequently found to be
elevated in asthmatic patients ?
A)IgA
B)IgE
C)IgM
D)IgD
Headache types

GT
provoked
unprovoked
3
m
6

https://t.me/SLPESTUDY
1) Which of the following have means that individuals have the
capacity to understand basic health information to make
appropriate health decisions ?
A)Health literacy
B)Cultural competency
C)Communication skills
D)Professional communication
2) Which of the following is an inhibitory neurotransmitter ?
A)Aspartate
B)Sertonin
C)Glutamate
D)Gama aminobutyric acid (GABA)
3) Who is responsible to promote the health in Saudi Arabia?
A)Saudi Food and Drug Authority (SFDA)
B)Ministry of Education (MOE)
C)Ministry of Health (MOH)
D)Health Colleges
4) Which of the following vaccines is contraindicated in a pregnant women?
A)Polio vaccine
B)Influenza (inactivated) vaccine
C)Measles ,Mumps ,Rubella (MMR)
D)Tetanus ,diphtheria ,Pertussis(T dap)

5)Which of the following viral infections best describes chronic infection?


A) Infection in which host cell dies due to interference with normal cellular function
B) Infections in which there is uncontrolled growth or immortalization of the host cell
C) Infection in which virus persists within the host cell but new virions are not
synthesized
D) Infection in which virus persists within the host cell and continues to replicate
the virions

6) MOA AND class Fluorouracil,-5-FU" ??


(Adrucil) (PYRIMIDINE ANALOGANTIMETABOLITES)
These agents inhibit pyrimidine synthesis during S phase; an active metabolite (F-
UMP) is incorporated into RNA to replace
uracil and inhibits cell growth, while another active metabolite (5-dUMP) inhibits
thymidylate synthetase.

7) Which of the following drugs is considered as an enzyme inhibitor that may effect
the metabolism of other drugs?
A)phenytoin
B)furosemide
C)cimetidine
D)theophylline

8) Which of the following parts of the body does urea synthesis take?
A) Liver
B)Heart
C) Blood
D) Kidney
9) Which of the following is the interpretation of a P-value of 0.05?
A) There is 0.05% probability that the result are due to random chance
B) There is 5% probability that the results äre due to random
C) There is 50% probability that the results are due to random
D) There is 95% probability that the results are due to random

10) Which of the following local organization's primary mission is to the


safety and quality of healthcare in health institution?
A) Saudi central board for accreditation of healthcare institutions
(CBAHI)
B) Saudi Commission for Health Specialties (SCFHS)
C) Saudi Food and Drug Authority (SFDA)
D) Saudi pharmaceutical Society (SPS)

11)Which of the following is a common enzyme polymorphism that


could potentially alter clopidogrel efficacy ?
A) Cyp2c7
B) Cyp2c9
C) Cyp2c10
oD) Cyp2c19
12) A 10-year-old boy is brought with complaints of fever sore throat
and non-productive cough for 1 day. Which of the following is best
regimen for non-productive cough?
A. Codeine 10 mg 4 times daily
B. Guaifenesin 50 mg 4 times daily
C. Dextromethorphan 15 mg twice daily

O
D. Dextromethorphan 30 mg twice daily
13) A 45-year-old patient with atrial fibrillation is treated with
amiodarone 400 mg/day and rivaroxaban 20 mg/day with the evening
meal. He has concern about his drugs routine monitoring.
Which of the following would be the best recommended regarding
amiodarone monitoring?
A. Slit-lamp examination at baseline and annually
B. Renal function tests at baseline and annually
C. Cardiac enzymes at baseline and annually
D. Liver function tests at baseline and every 6 months
O
15)- Which of the following vaccinations needs to be proofed for persons
working in a healthcare setting?
A. Tetanus
B. Varicella
C. Rota virus
O
D. Hepatitis B

16- Which of the following dosage regimen is approved for severe


Alzheimer's disease?
A. Donepezil 23 mg daily
O
B. Memantine 5 mg daily
C. Rivastigmine 6 mg twice daily
D. Galantamine 12 mg twice daily

17) Which of the following vaccines is specifically indicated for all


pregnant woman during each pregnancy, in addition to influenza vaccine?
A. Hepatitis B vaccine
B. Hepatitis A vaccine
C. Pneumococcal vaccines

O Tdp
D. Tetanus, diphtheria, pertussis.

18) A 36-year-old man not known to have any medical illness diagnosed
recently with an active pulmonary tubercolusis.
Which of the following is the best regimen to start?
A. Rifampin alone
B. Isoniazide alone
C. Isoniazide plus rifampin
D. Isoniazide, rifamnin. ethambutol and pyrazinamide
19)- Which of the following counseling points is most important to discus
with a patient who is taking carbidopa/levodopa?
A. Should be taken on an empty stomach
B. High-fat meal may increase the absorption
C. Should not be taken with calcium supplements
D. High-protein meal may decrease the absorption
O
20)What is the antidote for heparin toxicity?
A. Protamine sulfate
B. Sodium salicylate
C. Naloxone
D. Glucagon

21)- Which anti-hyperthyroidism drug could cause agranulocytosis?


A. Xylocaine
B. Metoprolol
C. Sodium lodide
D. Propylthiouracil

22-Which of the following type of water should be used for reconstitution


of parenteral preparations?
A. Sterile water
B. Distilled water
O
C. Sterile water for injection
D. Bacteriostatic water for injection
swfI
23)Which of the following drugs require extra precautions during handing to ensure
medication safety?
A. Methotrexate
B. Potassium
C. Dopamine
D. Thiamine

24) Which of the following medications is used to treat Alzheimer's


disease?
A. donepezil
B. amantadine
C. trihexyphenidyl
D. carbidopa-levodopa TPD
25) Which of the following food supplement is rich in omega3 fatty
acids?
O
A. fish oil
B. palm oil
C. peanut oil
D. coconut oil

26)Which of the following is another name for a true solution?


A. Heterogeneous mixture
B. homogeneous mixture
C. suspension
D. colloid

27) Which of the following is alternative name for vitamin B12?


A. biotin
B. thiamine
C. pyridoxine
D. cyanocobalamin

28)Which of the following is the expected bioavailability (F) of drugs


given by intravenous route?
A. less than 25%
B. 25%-50%
C. 75% -100%

O
D. 100%

29) A man patient is placed on a new medication to control his elevated BP. After one
month he noted that his breasts have become enlarged and tender. Which of the
following medication is the most properly prescribed?
A. acetazolamide
B. chlorthalodone gynecomastia
C.spironolactone
D. hydrochlorothiazide

30) Itraconazole capsules require acidic pH optimal dissolution and


absorption. Which of the following is an appropriate advice to optimize
the oral absorption of the drug?
A. take it with food
B. take it on an empty stomach
C. take it with full glass of water
D. take it with a proton pump inhibitor
UF H
31)Which of the following statements describes unfractionated heparin?
A. enolic acid derivative
B. propionic acid derivative
C. acetyl ester of salicylic acid
O D.mucopolysaccharide polymers
3
32)- Which of the following is the most practical method for sterilizing
ophthalmic solution?
A.autoclaving for 15 minutes
x
B. autoclavifig for 30 minutes
C. membrane filtration through 0.2-
D. membrane filtration through 5-

33) Which dietary supplement can be suggested to overcome jetlag?


A. Melatonin
B. Echinacea
C. Yohimbine
D. S-adenosyl-L- mwthionine (SAMe)

34) Which of the following is most likely to be associated with


heartburn?
A. paracetamol
B. chlorpheniramine
0C. potassium chloride RCL
D. magnesium/aluminium hydroxide

35) taken with food?


A.Apixaban

O B.rivaroxaban
C.dabigatran
da
36)most direct anticoagulant that causes dyspepsia? pi
A.Dabigatran
O B.rivaroxaban
C.apixaban
D.betrixaban
che
g
37) medication done in a vertical laminar hood?
A.Doxorubicin

38)-what's the medication contraindicated in CrCl less than 20?


o
A.Spironolactone
B.furosemide
C.metolazone

39) Medication for osteoporosis in menopausal women given


monthly?
A- Zoledronic acid
B- ibandronate
C-raloxifene

80mg PO
40- Pt. On furosemide 40 iv change to oral?
(2mg po =1 mg iv

41) what is the Clozapine test monitoring ?

o
A - Neutropenia (ANC) ~
8 - Liver Functions
B
C- Kidney Functions
D-Testing Of Vitamin 812 Levels
B Schilling
42)The authors report two isolated gastro- intestinal
tuberculosis in renal transplant recipients that illustrates the
difficulty of making this diagnosis and a brief review of the
literature on its clinical presentation, diagnosis and therapeutic
approach. What is the type of study in the above-mentioned
abstract?
O
A- Case-Series
B- Cohort Studies
C- Case-Control Studies
D- Cross-Sectional Studies

43)Ig cross placenta?


A. IgE
B. IgA
C. IgM
D. IgG
G
O
44) The cause of diphtheria ?
A. Pneumococcal
B. Bacillus
C. Staphlococcus

O
D. Corynebacterium Diphtheriae
:

: which of the following strong CYP3A4 inhibitor with grapefruit ?

1 - morphine

2 - Clopidogrel

3 - simvastatin

4 - ketoconazole

2 - Which of the following is least interaction with grapefruit ?

1 - Atorvastatin

2 - Simvastatin

3 - Pravastatin right answer

4 - Lovastatin

3 - Pharmacovigilance at which stage ?

1- stage 1

2 - stage 2

3 - stage 3

4 - stage 4

4 - Flash tablet means ?

1 - contolled release

2 - extend release

3 - Fast release

5 - Case scenario : patient takes Furosemide and gentamicin and amlodipine and melatonin , what
interaction would patient have ??

Furosemide and gentamicin cause severe ototoxicity


6 - Female Patient on beta blocker. Her BP is high and depressed, we need an alternative. She wants
to conceive, what is a comparable safe option?

o Methyldopa

7 - Patient is not absorbing B12, what kind of test to detect that?

o schilling test

Ig

IGE

What is the most Ig in the blood serum concentration?

IGg

TB which of the following TB drugs cause blurred vision ?

Isoniazid

Rifampin

Ethambutol

Pyrazinamide

Which cephalosporins are not active against Pseudomonas species - - Cefazolin

- cefepime

- Ceftazidime

( Lice )

Permethrin

Which anticancer take it once a week ?

Methotrexate

If a pregnant came to you and ask for complementary medicine for her N/V, which one?

Ginger capsule
Which infection need monitoring of resistance?

tuberculosis

Penicillin allergy

Lactam ring
mN
1
Sulpha allergy which structure is safe ?

Sulpha group

Efficacy

Cost minimization analysis

Efficacy

Cost effective analysis

Which combination with statin cause severe rhabdomyolysis ?

With gemfibrozil

Discontinue clopidogrel before surgery ?

5 Days

Decrease warfarin level ?

Multivitamin

Ceftriaxone dose in bacteria meningitis is 4 g/day IV divided every 12 to 24 hour ; MAX 4g/day

g BID

2g BID
Contraindicated in TIA or stroke : prasugrel
Which of the following insulin is the most long acting ?

Degludec

Glargine

Side effect of Niacin ?

give ASA
Flushing and dizziness

Aspartame?

Sweetener

Drug devesion ?

medical and legal concept involving the transfer of any legally prescribed controlled substance from
the individual for whom it was prescribed to another person for any illicit use.

Gray baby syndrome ?

Chloramphenicol

Brown urine ?

Nitrofurantoin

Transportation ?

Direct non medical cost

Drug increase milk ?

Metoclopromide
D antagonist
Valerian plant uses ?

Used as sedative
Quality adjusted life year ( QALY ) ?

1 = perfect health

the process of decreasing the particle size and grinding solid with liquid known as ?

Levigation

Beta carotene?

Retinol ( A )

Vaccine can be frozen ??

Varicella zoster

USP non sterile ?

795

Which of the following drugs cause diarrhea?

Morphine

Magnesium oxide

Omeprazole my
Antidote of Metoprolol and amlodipine ?

Glucagon BB cab

When MCV indicated at which age ?

9 month first time second time 12 month

Which of the following not found in Gram positive bacteria ?


or
Lipopolysaccharide
Over dose of warfarin cause ?

Constipation

Diarrhea

Gum bleeding

P.C means ?

After meal

PRN means ?

As needed

Unit of Cost effective analysis?

Natural units

elderly
Eldery female 74 years needs complement calcium supplement ?

1200mg

Elderly man 64 years needs complement calcium supplement?

1000mg

Which of the following is an active form of vitamin D ?

1,25-Di-hydroxy-cholecalciferol

Sar nalidixic acid ( Quinolone ) which part is pharmacophore ?

What resource should be used when searching for the most current clinical trials on the use of novel
anticoagulants for pulmonary embolism?

a) Pharmacotherapy, the pathophysiologic approach

o
b) PubMed
e
First order

less than
C
Zero order

Question about lipinski role 5


u
fess b
10 HBA
Log pH
Cacluation

1 - Crcl male , female


5
2 - Half life

3 - Vd

4 - Cl

5 - Infusion rate

6 - BMI

7 - Specific gravity

8 - Therapeutics index

9 - Bioavailability

Regulation& ethics , calculation , study design

Study design

Assertive , passive and aggressive

valid Controlled medication days


UTI

which of the following drugs used type 1 diabetes ?

Empagliflozin or canagliflozin

Patient with 3.5 INR and he did mitral valve before ?

No intervention

Rx Rx
Calculation
Crcl (140-age) x weight/ 72 x Scr if female (x0.85)
VD
Vd Vd Cl the
Dose/C OR
K/Cl
dz L
T1/2
0.693/K
K= Cl x Vd
Total Cl
Vancomycin equation

Structure
Benzodiazepines (7 member ring fused to one benzene ring, and another
benzene ring)
Celecoxib (5 member ring with 2 N + 3 F ions + sulfonamide)
Aspirin Must contain acid (COOH)

DOC for HTN with diabetes ACE


or ARB
(S) for strength
Total doses of Hepatitis B vaccine 3 doses

I
Nausea in pregnancy: ginger&VitB6
Vaccine CI in pregnancy: Varicella

Who is responsible for the shortage of drugs in Saudi Arabia SFDA


Patient with gout & MI and colchicine isn t effective what to give
drug increase constriction of ductus arteriosus in neonate NSAIDs
(Indomethacin)

drug increase the risk of peptic ulcer NSAIDs

Enzyme catalyzes the transportation of cholesterol to the cell LCAT

was not in option (spironolactone or hydrochlorothiazide)


CT
how many kilocalorie in 1 gm of dextrose 3.4 kcal
J
case patient with gout and which drug is contraindicated: furosemide

prophylaxis antibiotic for colorectal surgery Metronidazole,


erythromycin
antiepileptic drug have the risk of both SJS AND TEN Carbamazepine
and lamotrigine
sevelamer indication
used in Type
control of serum phosphorus

what is the polymorphism cause cough in ACEI: Accumulation of


bradykinin
Woman with PE on warfarin want to be pregnant: Enoxaparin
Oral anticoagulant we should give IV anticoagulant for 5-10 days before
use Warfarin ✅ Dabigatran 
✅ Edoxapan 
drugs contraindicated in G6PD deficiency Sulfa drugs ex. Bactrim
0for
autoin
what is related to pharmacokinetics : bio transformation
Antidote of opioids Naloxone
Patient came to you with vomiting and sever pain in right lower
abdomen, what to do? Refer to hospital
plate method: DM
HTN diet
increase zinc
increase potassium
decrease zinc
decrease potassium
What the the indication of amitriptyline
Nausea
Vomiting
Headache
Neuropathic pain

Patient with hyperthyroidism on methimazole , but still have


palpitations
1. increase dose
2. Decrease dose
3. give anothe agent for symptoms relief
4. no change
BB
patient talking with pharmacist and the pharmacist made eye contact
verbal
non-verbal
visual
the pharmacist offer handshake to the director,type of
communication
verbal
non verbal
physical
doctor order 0.2%(drug)for atopic dermatitis with very dry skin, what
make
cream
solution
ointment
suspension
what is the coating of enteric coated tablets
methylcellulose It Pmcp
And other cellulose options
Definition
Budget
A budget is an estimation
of revenue and expenses over a
specified future period of time and
is usually compiled and reevaluated
on a periodic basis.
Budgets can be made for a person,
a group of people, a business, a
government, or just about anything
else that makes and spends money.
Accounting:
Systematic recording of information
which involves analyzing, classifying, summarizing
and interpreting business transactions
Prevalence:
The number of existing cases of a disease in
a population at a given time.
Incidence:
The number of new cases of a disease or disorder in
a population over a period of time.
Types of planning

O
Which of the following use in central line ?
A- D5w
B- 0.9 NaCl
C- 3% NaCl
D- D10w

best treatment for st.aureus positive mastitis ?


Cephalexin

57-year-old man with liver cirrhosis and ascites and a Child-


Pugh
class score
of 8 is having severe pain due to injury. What would be the
recommended
medication?
A. Codeine
B. MorphineC. Diclofenac
D. Acetaminophen
O
56- Which of the following is the result of drug interactions
between
birth control pill and erythromycin?
A. The effectiveness of erythromycin may increase
B. The effectiveness of erythromycin may decrease
C. The effectiveness of the birth control pill may increase
D. The effectiveness of the birth control pill may
decrease

Observing evidence of instability during compounding is an


important
step.
Which of the following is happened should have discarded
the product
and not dispense it ?
O
A. Color intensity r
B. Weight Changing
C. Size of the holder
D. Chemical potency

Drug choose to diabetes type 1


Glipizide
Rapeglinide
Nateglinide
Empagliflozin

Antidote for apixaban?


Andexanet Alfa

condition treated without prescription agent?


A. Vertigo
B. Water clogged ear
C. Tinnitus
D. Temporary hear loss

-Which of the following should be avoided in elderly?


Betablocker

Which IV container need filtered needle !?


A.vial
B.bag
C.glass ampoule
D.all iv preparation need
Q6: Dose depends on what (Levodopa/carbidopa)?
A. Levodopa alone
B. Carbidopa
x x
C. Both Levodopa and carbidopa

Q7: Dose depends on


(Trimethoprim /Sulfamethoxazole)?
A. Sulfamethoxazole
B. Trimethoprim
C. Both Trimethoprim and Sulfamethoxazole

COPE limitation ?
- system downtime
- System error
- System consumes time r
- System dose not reduce error

Patient comes with dark urine, pale stool , yellow eyes why
?
- elevated albumin
- Elevated bilirubin
7
- Elevated triglycerides
Osteomyelitis duration?
- 1-2 weeks
- 4-6 weeks
- 7 days
Which of the following is no adherence factor od dm pt
taking
insulin?
ro-- poor
z
injection technique
Pt afraid from needles

Which of the following contraception method has the


highest estrogen
and is contraindicated in women with history of stroke ?
- vaginal ring
- Patch
- Progestin implant copper iud
Manager wants to remove medication and wants to
formulate a
committee to assess and evalute he should assign who ?
- planning
Eta
- Organizing
- Controlling
- Leading
Q- Longest half-life ? lgG
Q- Immunoglobulin found in serum predominantly ? lgG
Q- Strongest Ig? IgG
Q- Ig on the blood ? IgG
Q- Ig have gamma globulin? IgG
===============
Q- Frist Ig antibody appear ? IgM
Q- New infection ? lgM
Q- Frist release in body ? lgM
Q- Immunoglobulins cells found in colostrum ? IgA and
IgM
Q- Fastest IG ? IgM
Q- IgM
================
Q- IgA
Q- Ig in milk? IgA
Q- Ig in serum, saliva and GIT ? IgA
===========
Q- Hypersensitive lg fond in fat tissues ? IgE
Q- Ig in asthma? IgE
Q- Angioedema type of hypersensitive ? lgE
Q- Peanut allergy symptom ? Type 1(lgE)
Q- Penicillin allergy ? IgE
Passive immunity immune IgA and IgG
Q1 / 12 years girl with rash from oxcabamazepin change to ?
Ans :
Shift to topiramate or ethosuxamide.
Q2/ Dose of carbidopa/levedopa depend on ? Ans :-
carbidopa
e alone levodopa
Q3/ Case taking medication then added prazocin suffer from
postrual
hypertension ? Ans : stop prazocin and shift to alfuzocin)
Q4 : Long case with test baljet test ++++ ?
( digoxin - ....)
Q5/ Drug stop breast milk ?
( metoclopramide - equal quality contraceptive tab ....)
Q6/ Mastitis treatment ?Ans : cephalexin or dicloxacillin
Q7/ Calculate the Enoxaparin dose for weight 112 kg Per
one dose ?
ans 168
1,5mg /kg/ od or 1mg / kg/ bid .
( 110 - 160 - 180 ..)
Is
Q8 / Side effect of oral corticosteroids ?Ans :
Candidiasis
Q9/ Question about role of protease enzyme ? Ans :
proteins
Q10/ Renal failure patient need diuretic ? Ans : furosemide
Q11/ Which one Rate control drug ? Bb, ccb , digoxin
rate control aniodarone
Q12/ Drug containdicated with verapamil ?
( digoxin - ...) CCB quite digoxin
Q13/ Element for bariatric surgery ?
( selenium - ...)
Q14/ Case patient taking Lisinopril must wait before chang
to
sacubitril/valosartan ARNI ? Ans : 36
Q15/ Topoisomerase ligate ? Ans: 2DNA
Q
/ 16

TogII
Atorvastatin + rosuvastatin +fluvastatin
or A
fluvastatin 2pm
fluvastatin

Q17/ Planning (1-5) years using resource for achieve goals ?


Ans :
resoj.es planning
organizational planning
Q18 / Trigonella is available more in ? Ans : fouengrek
I
Q19/ Drug increased lacrimal, gastric secretions ? Ans
acetylcholine Ach
Q20/ Hashimotos symptoms ? ans : wt gain
Q21/ Drug less lipophilic ? Ans : atenolol
Q22/ Vincristine dispense ? Ans : biggy bag

Q23/ Anticoagulant for atrial fibrillation and mechanical


valve ?

( rivaroxaban - dabigatran - warfarin)

Q24/ Anticoagulant take with food ? Ans: dapigatran Rivaroxabat


Q25/ Role of ciprofloxacin + metronidazole in treatment of
crohns ?
Ans : mild
J
Q26/ Antimetabolite drug cause bleeding ? cystitis
Ans : Cyclophosphamide hemorrhage
Q27/ Mcv4 vaccine is given at which age ? Ans :9 Months
Q28/ Screening program by ministry of health to student to
discover
diseas
( dental - obesity - vaccination ...etc ) to detect it is
considered ?

( primary care prevention - school health programs


Which of the following counseling points is most important
to discus
with a patient who is taking carbidopa/levodopa?
A. Should be taken on an empty stomach
B. High-fat meal may increase the absorption
C. Should not be taken with calcium supplements
D. High-protein meal may decrease the absorption

A 45-year-old patient with atrial fibrillation is treated with


amiodarone 400 mg/day and rivaroxaban 20 mg/day with
the evening
meal. He has concern about his drugs routine monitoring.
Which of the following would be the best recommended
regarding
amiodarone monitoring?
A. Slit-lamp examination at baseline and annually
B. Renal function tests at baseline and annually
C. Cardiac enzymes at baseline and annually
D. Liver function tests at baseline and every 6 months

60-year-old man with 6-year history of myocardial


infarction and
gout. He presented to the clinic complain of severe pain in
his first left
consistent with acute gout flare. He mentions that he can't
colchicine.
What is the most appropriate pharmacotherapy
recommendation for
his
acute flare-associated pain?
A. Ibuprofen
B. Allopurinol
C. Indomethacin
D. Acetaminophen
A drug has a volume of distribution of 35L in a 70 Kg man.
Which of the following best describes Its distribution?
A. it is bound to DNA
B. it is dissolved in lipids
C. it has low bloavailability
D. it is mostly distributed in plasma

Which detoxification pathway is active paracetamol


e
toxicity?
A. Oxidation
B. Reduction
C. Conjugation with glucuronide

O
D. CYP-450 dependent glutathione

Which of the following herbal products can potentiate the


effects of
oral
antidiabetic?
A. Garlic
O
B. Ginseng
C. Green tea
D. Echinacea

suspended agent is important for preparing an oral


suspension. Which
of
the following is the characteristic of this suspending agent?
A. Hydrophilic, to inhibit agglomeration
B. Hydrophobic, to inhibit agglomeration

G
C. Hydrophilic, to prolong gastric emptying time r
D. Hydrophobic, to prolong gastric emptying time

Oxytocin,:
high doses may lead to hypertension
B. steady state is reached within 5 to 10 minutes
C.uterine hyperstimulation is an associated adverse
effect
D. higher doses can lead to vasopressin receptor related
urinary
Retention

When will we start TPN for the patient not intake orally
A- After 7 days
B- within 3 days
C- within 1 day

Attenuated Mean?
A. Active virus
B. Killed virus
C. Inactivated virus

55- Which of the following is the benefit from adding a


conjugate of
polyethylene glycol (PEG) in some medications like Peg-
interferon?
O
A) to extend the half-life of medications
B) B, to extend the shelf-life of medications
C) to enhance the effect of the medications
D) to enhance the elimination of the medications

Q1-sign of iron deficiency anemia ?


- koilonychias
w
Q2-steven johnson syndrome caused by ?
Carbamazepine and lamotrigine

Which of the following should not be done for sound alike


medications? t
A. Use tall man letter
B. Verbal order
O
C. Non-verbal order

contraindicated with
neonate

Paracetamol
Ciftriaxon
Cefalexine

Newly graduated pharmacist hired in multinational... what


would be his access of care?
A. General
B. Limited
Which one if contraindicated in second and third trimester
due to fetal
renal failure? A CE l
S.A is 65 years old came to hospital with dyspnea and
shortening of
breath by asking him he had this limitation with regular
daily
activities like brushing teeth he also had to put 2 pillows
under him
during night (pulmonary edema) What is the class of H.F
according to
NYHA
Answer
a- I
b- II
o
c- III
d- IV

Drug with low bioavalability?


Vancomycin

Septic shock treatment?


Fluid then NE

uti female with penicillin allergy?


nitrofurantoin(1st)
cefuroxime(2nd)

Which patient population are at high risk of medication


errors?
A- Pediatric
B- Aldolescent
C- Adult
D- Childbearing age women
Before drug be approved we see the cost of safty and non
safty which
type of cost:-
Cost of illness
Cost of benefits
Cost of effectiveness
Cost of utility

a drug can prevent death from highly fatal disease but can
not cure it.
What happens?
Increase incidence
Decrease incidence
Increase prevalence
Decrease prevalence
monitoring :

Xa factor for enoxaparin


INR for warfarin
APTT for heparin
Strong evidance study ?
Rct

Glaucoma treatment in asthma patients?


Latanoprost + betxolol

Essential HTN , pulmonary edema ?


A - Propranolol
B - frusemide
C- indomethacin
D - captopril
indapamicle
When we start adjusting doses for renal imapirment? 100-
90-70-30
30

Dose of caffeine in neonate where to look?


- drug in pregnancy and lactation
- Hospital formulary drug
- Harriet Lan Handbook

Group of Patients taking methadone and giving drug A


which cause
miosis and decrease withdrawal symptoms and drug Bwhich
caus
mydriasis and cause withdrawal symptoms and drug C
which inhebit
withdrawing symptoms,what is drug A,B and C?
A morphine B naloxone C methadone
C
DARE
dihydrofolate reductase inhibitor

?
- Adenosine
- Guanine
- Cystine
- Thiamine i guess
Ig
1-IgE
2-IgG
3-IgM
dumping phenomenon occure in ?
- Modified release
- Soft gelatin capsules
MR
- Hard gelatin Capsule
- Cobressed tablet
Treatment of central line-associated bloodstream infections
(CLABSIs) ?
- Gentamicin
- Vancomycin
- Ceftazime
- avibactam

Trandermal
Patch ?
Et
Which one is bot phisyochemical characteristics of

Non ionic
Need High melting point
Molecular size less than 500 d

O
High dose
HF r EF
Patient diagnosed with HF with ejection fraction 35% what
to give?
Spironolactone 12.5 mg no r
y benefit
Fursamide 40mg bid
What is the acceptable bioavailability in self life shelf life
98%
0
97%
90%
87%
Role of NAD/NADH?

O
A. Electron transfer
r
https://t.me/SLPESTUDY
Ig in Type 2 ?
A. IgG
e
severe Crohn's disease?
A. IV cyclosporin
Tv Csr
Treatment of amoetba ?
A. Ipecac
Increase Prolactin: Decrease
Dopamine (DA)
Decrease milk production? Bromocriptine jaangionist Metoclopramide

Increase milk production?


Cabergoline Methyldopa
Estradiol
Serotonin
I
GABA
Best advise for chewable aspirin tablet? Opioid
only chew
Pregnant lady with symptoms of eclampsia, what to give?  Magnesium sulfate 

Patient on warfarin and comes to clinic and the rustle of INR pending what to ask the patient?
A. Gum bleeding

Siglec-8 is an inhibitory receptor and studies have shown that engagement of the receptor results
in inhibition of mast cells and apoptosis of eosinophils.

se
Disopyramide not used eldery — exacerbation of heart failure.
te
e
- Erythromycin eye drop In neonates — Gonococcal eye infection refers to infection of the eye
caused by the bacteria Neisseria gonorrhoeae, a sexually transmitted infection. Newborns typically
become infected.

Structure of the health care system in Saudi Arabia?


3 tiers
Which of the following measurements is used to assess patient adherence ?
Med Event
K
A)Drug Utillization reviews

According to the FDA categories of the drug safety during pregnancy Amoxicillin is under category

I
B
Animal studies show no risk to the fetus and no controlled human studies
What does that mean ?

Newly graduated pharmacist hired in multinational. what would be his access of care?
A. Loses access to public
B. Doesn’t lose access to public
C. Can get healthcare from certain private hospitals

E
FlashTab technology is used in making tablets means
Fast release

Histamine action? Stimulate gastric secretion

- a patient came to community pharmacy and said you sell the cosmetics in higher price than the
others and pharmacist said yes we are selling them in higher price because they are original?
assertive

How to reduce UTI risk in female? Urinating after sexual intercourse?

improve physical endurance? ginseng

Ceftriaxone dose in bacteria meningitis — 2 g BID

Non-pharmacological treatment dysmenorrhea? topical heat

for 7 days
J
2 years old with sever otitis media ? Amoxicillin

UTI pt with e coli culture media sensitive to all antibiotics. Which one will you give?
NitroForan
What to do to increase acidity of the urine : drink cranberry juice or probiotic yugert

Vaccine given to baby before hospital discharge : hepatitis B


live and inactivated vaccine be given together at the same time

which of the following is considered a moderate physical activity ? Brisk walking

Disadvantages of RCT? A. expensive

Patient started on amiodarone where should you refer him to?


A. pulmonology
B. ophthalmology
C. psychologist

patient came to you and you as a pharmacist find his BP 175/90 what to do ?
A. tell him to see HCP within the week
B. ask the nurse to triage and book for him an urgent appointment with his physician
O
Women with inflammatory acne asks pharmacist about cleansing product what will you

Talysilic
recommend?
A. hydrocort 1% acid
B. mild facial soap

structure of ergot alkaloid and asked which of the following ergot is used for migraine?

oA. dihydroergotamine
B. and others choose were similar to ergotamine but with different spelling

Grapefruit and statin interaction?


Siwa statin
Duration for anticoagulants provoked DVT? 3 months

Duration of anticoagulants unprovoked DVT? 6 weeks


2 6 months
Patient have allergies from

Fingolimod and pregnancy? Use contraceptive during and 2 month after discontinue

Which characteristics of sustaind release dosage form? A. large dose


B. 4-8 hr biological half life
C. high solubility

Monitoring of warfarin ? PT/INR


NRU
QALY 1 means ? Perfect health

Health coverage for pilgrims?


A. Pilgrims have no access to publicly healthcare

O
B. Non-Saudi pilgrims must have health insurance before coming to Hajj

Patient comes with dark urine, pale stool, yellow eyes, why?
A. elevated albumin
B. elevated bilirubin
O
C. elevated triglycerides

Which of following interactions with clopidogrel and increase risk of bleeding?

Which statin should be taken at evening? Simvastatin 

A patient came to satellite pharmacy in ER and says his is out of insulin, what you should do?
A. Tell him to buy from retail pharmacy
B. give him one vial
C. tell him you cant give him without prescription

O
D. tell him to go to ER to assess his case

Best advise for chewable aspirin tablet?

0A.B. Should
Should be chewed
be swallowed
C. Can be Chewed or swallowed
D. Should be crushed
D. Should be crushed

The effects of four anti-hypertensive drug classes (A-D) on the Renin-angiotensin system are
shown (see table)
Which class is represented by the drug -C?
A B C p
A . Diuretics (D)
B . Direct Renin inhibitors (A)
C . Angiotensin II receptor blockers (C)
D . Angiotensin converting enzyme inhibitor (B)

Which of the following is the disadventage of conducting randomized controlled trial ?

O
A)Expensive
B)High possibility of bias
C)Difficult to control confounders
D)Cannot be used for hypothesis testing

According to the FDA categories of the drug safety during pregnancy Amoxicillin is under category
B
What does that mean ?
A)No adequate animal or human studies have been conducted
B)Controlled human studies show no fetal risks from amoxicillin
C)Evidence of human fetal risk exists ,but benefits may outweight
D)Animal studies show no risk to the fetus and no controlled human studies
B
Which of the following best describes the structure of the healthcare systems in Saudi Arabia ?
A)Primary care based
B)Private based care
C)3-tier system
D)2-tier system

ADE
Which of the following is reguired to consider an advers drug event report a valid report?
A)Drug dose
B)Patients tall
C)Event name

C
D)Reporter’s phone

Which of the following inhibits gluconeogenesis?

O
A)Insulin
B)Glucagon
C)Epinephrine
D)Glucocorticodes

There are four principles in biomedical ethics these ethics autonomy justice, Beneficence ,and one
more principles Which of following as the fourth principles of biomedical ethics?
A)Privacy
B) Dilemmas

O
C) Non-maleficence
D)Conflict of interest

cough
A 3-year-old girl with symptoms of caught for the past 2 days came to the clinic with her
mother .She has no fever and not other symptoms .Which of the following is the appropriate
recommendation ?
A) Vicks
B)Honey
C)Diphenhydramine
D)Dextromethorphan

Which of the following method is successful in reducing medication error and enhance patient
safety in hospitals?
A) Performing continuous education sessions on safe dispensing
B) Providing the pharmacy with the most updated drug references

o
C) Implementation of computerized physician order entry (CPOE) system
D) Encourage medical staff to participate in conferences and scientific meetings regular

Which of the following substances is used to disinfect gloves during compounding sterile products?
A) liquid soap
B) 70% isopropyl alcohol
C) sodium hypochlorite 5%
D)sulfuric acid and nitric acid (3:1)
D)sulfuric acid and nitric acid (3:1)

I
which of the following anticoagulation agents the highest risk of causing dyspepsia? dabigatran

An adult female patient with weight 80 kg and 160 cm height How do you categorize this patient
weight based on the following classification:

727
3125J
A. obese
0B. normal 8
C. overweight pmI
D. underweight

Which of the following medications to be used under caution is in patients with ischemic heart
disease?
A. lisinopril
B. celecoxib
0C. metoprolol
D. simvastatin

Which of the following is an important counseling point for woman starting on fingolimod?
A. Use effective contraception to avoid pregnancy during and 2 months after discontinuing
treatment
B. Pregnancy should be avoided for 6 months after discontinuing treatment
C. Use effective contraception to avoid pregnancy during treatment
D. Fingolimod is safe during pregnancy

Which of the following vaccines is recommended within the first twelve hours of birth?
A. Varicella
B. Hepatitis A
C. Hepatitis B
D. Measles, mumps and rubella (MMR)

Which of the following vaccines is recommended to be given at birth in Saudi Arabia?


A.Rotavirus vaccine
B.Hepatitis A vaccine
OC.Hepatitis B vaccine
D.Haemophilus influenza type B vaccine

Which of the following is high intensity of statin ?


A. Atoravastatin 10
A tow 40 80 ng
OC. Simvastatin 10
B. Atoravastatin 40
20 40
D. Simvastatin 80
Rosuva ng
Which of the following medications for osteoporosis will significantly reduce the risk of hip fracture?
A. Phosphate
B. Risedronate
DC. Raloxifene bisphosphonates
D. Calcitonin

Which of the following drugs is contraindicated in heart failure?


A. Captopril
CF
oC. Furosemide
B. Celecoxib

D. Amiodarone

An 18-year-old obese girl with newly diagnosed depressive illness is being advised to start
antidepressant treatment. Which of the following is best to initiate?

OA.B. Paroxetine
Bupropion
Lwt
Awt
C. Mirtazapine
Awt
D. Amitriptyline
Awt
Aspirin is well-known antiplatelet aggregrgating agent(see image)
Aspirin

What is the role of the acetyle group in its antiplatelet activity?


A.it represents a good leaving group to form the active salicylic acid
B.it provides hydrogen binding with tyrosine moiety on COX binding

OC.D. itit provides


provide covalent binding with serine moiety found on COX binding site
hydrophobic binding with a tryptophan moiety on COX binding site
Which of the following statements best describe drug dependence?
A. Discontinuation of the drug will produce withdrawl symptoms
O
B. Action of one drug is dependent on the presence of another drug C. Discontinuation of the drug
will not produce withdrawl symptoms
D. Patients progressively require increasingly larger doses of drug to achieve the same effect

Which of the following solutions is required for the reconstitution of parenteral formulations?
A. USP standard sterile water
B. USP standard purified water
C. USP standard highly purified water
D. USP standard sterile water for injection
O sw f I I
Which of the following is a pharmacological action of dopamine?
A. elevation of blood pressure
B. paralysis of skeletal muscle
C. stimulation of gastric secretion Histamine
0D. activation of cardiac contraction
CO2
Which of the following is the most likely mechanism of carbon monoxide poisoning?
A. inhibits the gag reflex
B. paralyzes the muscles of the diaphragm
C.decreases the oxygen-carrying capacity of the blood
O
D. reacts withh amino acids in the body to form ammonia

-Which bio similar drug is approved for the treatment of moderate to severe allergic asthma?
A. Infliximab
B. Efalizumab

O
C. Omalizumab
3
D. Trastuzumab

1A 55-year-old man presents with an excessive bronchial and oconasal secretions. History revealed,
is exposed to an organophosphate insecticide. Which of the following is the best management?

I
A. Atropine IV
d by pralidoxine
B. Diphenhydramine orally https://t.me/SLPESTUDY
C. Pralidoxime intramuscularly
D. Physostigmine intramuscularly

Which one of the following side effects is associated with bisphosphates use?
A. headache
B. blurred vision
C. hypotension

o
D. osteonecrosis of the jaw r
Which of the following is the exact date for a drug to be safety taken if its expiration date December
2018?
A. December 1,2018
B. December 15, 2018

O
C.December 31, 2018
D. January 1st , 2019

4116 I
Ask which one is IV ?

w
O

Ask about example on moderate activity ?


e

What is name of these structure :


name of structure

Is
9

H
to
O

O
Ask about MOA action of Celecoxib ?
3f
sulfa

2 inhibitor
COX

PG inhibitor
c
Ask about physostigmine structure in comparing with pseychostigmine ?

https://t.me/SLPESTUDY
Ask about physostigmine structure in comparing with
pseychostigmine ?

Calculation:
BMI
CrCl
Absolute Bioavailability
Flow rate
Allegations
vaccination Igg igm
A,B,C,D
rash nicotine acid aspirin -tramdol- preslodone
hood air flow
Quinn diazepam ethics

clodprgel
high potassium

high alert anti thrombosis


Mechanism action. Of amphetamine
increasing the amounts L of dopamine, norepinephrine, and
serotonin (to a lesser extent) in the synaptic cleft

study cohort
cost benefit
vancomycin
cough tuberculosis

fentoyl
Morphine
chronic fugal family history

xf.tv
Trimethoprim / Sulfamethoxazole
resistance
amoxicillin

antidote
B3 niacin pellagra

https://t.me/splestudyplan

minimum age limit for giving levocetrezine

Oa. after 6 months


b. after 1 year
c. after 4 yrs

anaesthetic with higher chances of cardiotoxicity


a. nitrous oxide

Ob. isoflurane
c. ketamine
d. propofol

https://t.me/SLPESTUDY
which of the following decreases the effect of phenytoin

a. alcohol
ob. chronic renal toxicity
c. mild renal impairment

Patient just recovered from myocardial infarction 10 days ago and


discharged from the hospital, to prevent recurrence of arrhythmia which
medication is most suitable for him

verapamil
quindine
sotalol
Amidarone

A 75 yr old man on alzhimers treatment is taking rivastigmine 6mg and


experiences symptoms like
vomiting and nausea. What is the appropriate measures to be taken

Ob. take rivastigmine 2 hr before food


a. stop rivastigmine and substitute with donepezil

c. stop rivastigmine
d. take rivastigmine after food

Case about lady ingested a rodenticide in which lab reports were given.
Serum potassium seems to be low. Which among is possible
Options

zinc phosphide
aluminium phosphide
barium carbonate
Zn
aluminium hydroxide

Which characteristic of Benzodiazepines is a reason for increased


withdrawal symptoms
Options
longer half life of drug,

2.shorter half life


protein binding

patient is with dementia and depression has been


taking the medicines for the last few years. One day
when he went to supermarket and he suffered from phobia and he

treatment should be given to the patient


A. Fluoxetine

CB. Buspirone
C. Clozapine
Anxiolytic
D. Gabpentin

Narcotics
What will be the storage duration for NRX drugs in
Saudi Arabia
a) 3 years
b) 4 years

0d) 2 years
c) 5 years
lab result is given. Sodium, Potassium, Hemoglobin
and RBC levels are normal. The WBC level is low
compared to normal value. What will be the reason for
this

Ob) Hemolytic anemia


a) Infections dd WBC
c) Thrombocytopenia
d) Hypokalemia
e) Hypernatremia

Business Planning:
T
To determine the feasibility
Planning: most critical elements

Facultative anaerobes

o
Growth occurs in presence or absence of O2
Greater growth in presence of O2
s

Angioedema
by
AC EI
Omission : 2nd most common error in the medication use process.

IE
Wrong time 1st common error in the medication use process.

Duplication error 2 drugs from the same class are prescribed


What resource should be used when searching for the most current
clinical trials on the use of novel anticoagulants for pulmonary
embolism?
a) Pharmacotherapy, the pathophysiologic approach

O
b) PubMed
c) Uptodate
d) Micromedex

cl in neonate Ceftriaxone. Cause hyperbilirubin


Phase 1: Early rapid repolarization due to Na channel close

28) Which of the following medication error is difficult to prevent ?

g
A)Omission error
B)Wrong time error
C)Incorrect duration of treatment
D)Allergic reactions to an unknown patient

Biguanide (Metformin): Polycyclic Ovarian Syndrome (PCOS),


Decrease weight
Renin-Angiotensin-Aldosterone-System-Inhibitors (RAAs-I) :
A. ACE-I:
# SE: Hyper K, cough, Angioedema, hypotension
# Cause of cough: increase the of bradykinin
# CI: in kidney failure and pregnant woman (fetal growth)
# ACE-I it is used: to convert Macroalbuminuria to Microalbuminuria
# ACE-I should be taken to ALL patient with HF to decrease mortality
except if there is CI
B. ARBs & Renin-I (aliskiren):
# aliskiren CI in kidney and pregnant
# Both are less cough and angioedema
Beta-Blockers:
Propranolol it is used in: Thyroid storm, HTN, Anxiety, Migraine
Labetalol: use in pregnant with HTN
# ALL beta blockers are CI in: asthma, DM

Albuminuria (regardless to race and CKD): ACE-I or ARBs

Uricosuric:
Increase uric acid excretion which lead to decrease the uric acid Conc. In blood.
Ex.: Probenecid
Allopurinol dose depend on Crcl:
o Crcl 3-9: 100 mg /day
o Crcl 10-20: 200 mg / day

MSSA: ceftriaxone, daptomycin, oxacillin


MRSA: Vancomycin, lineside
line Zolid

Grapefruit NOT: with amiodarone

Pediatric: Tetracycline (tooth discoloration), Fluroquinolone (QT


prolongation)

Plant sources of anticancer:


Vinca Alkaloid: Vinblastine, Vincristine, Vindesine, Vinorelbine
# Vincristine Fatal if given intrathecal ONLY IV infusion
Digoxin decreases with antacid

Bisphosphonate (for OP) à Upright position + empty stomach (morning)


Alendronate weekly
Ibandronate:: Monthly
Zoledronic Acid
yearly
Patient with pneumonia received IV ceftriaxone and azithromycin, after
3 days he became afebrile with normal WBC level, what to do
A) Continue treatment 10-14 days
B) stoptheantibiotics

xC) Switchtooral
D) Stop azithromycin and continue ceftriaxone r
o y.o patients with reduced kidney function (CrCl 23) and multiple
disease, has urinary tract infection, what is the best antibiotic regimen
A) Ciprofloxacin 400 mg q12
B) Ciprofloxacin400mgq24
OD
C) Meropenemq8
D) Meropenem q12

TPN was clear then become slightly collid after administered to


patient what to do
stop - continue - slow the rate - warm it
ku
of all lower
what the meaning of Master Budget
aggregation
produced by a company
Budgets
Doxycycline consoling at what time morning? various functional areas
with food or water only ? Doxycycline take 1 hour prior to or 2 hours

two
after meals and take with adequate fluid
meals ) what the dose it mean if pt have high level CL and Low GFH
kidney
pop1
new technique drug stirle sold
implant
layered- implant
defforxamine
AAU hyperkalemia
digoxin toxicity test what will shows? Hypokalemia, hypomagnesemia
and hypercalcemia 99k drug r ca

esterify µon sterile


di
Review Article micro
fEV280
I
Gold stage 1 what the first line dose ? bronchodilator
Iron antidote? deferoxamine
master budget? cash I think .. not sure
budget definition revenew a
expenses
what miniral test shows digoxin toxicity
hypocalcimea , hyperphosphotemia , hypomagnisemia
99k
what in 3iry sources, ex
Simvastatin at what time? bedtime
corticosteroids asking about if it's play role as releving pain .. as an anti-
inflammatory effect

I
Hashimoto's disease what to give? levothyroxine
high level of Cl and Low GFR.. what that's mean
pt. with came with cough for 2 days only and history: controlled
hypertension taking enalapril, what to do? stop enalapril, switch to
ARBs, give him dextromethorphan

pregnant in her 30 week what vaccines she should take ? Tdap +


influenza
before using infliximab , what should be tested? Active and latent TB
infection; prior to and periodically during therapy..
codine to morphine by? CYP2D6
pregnant with hipB what to give her baby? hipB + immunoglobulin hipB
vancomycin trough level in sever infection ? 15-20
vancomycin dose depends on ? Trough test
pt. took a lot of med one of them codine and came with miosis and ..etc
after we give her naloxone .. what was the cause ? codine .. convert to
morphine
COPD fist line treatment ? Bronchodilator
quality adjusted life year ? Utility cost
SV node a specialized myocardial structure that initiates the electrical
impulses to stimulate contraction

vaccine taking orally and injection? Poilo


Treatment of UTI
dose of paracetamol? the maximum dose of paracetamol is 3000 mg
(3g) in 24 hours.

aaa
id
pt. 60 y with hypertension and diabetes CHA2DS2-VASc score ? 2
drug cause folic acid deficiency? Methotrexate
pt with osteoporosis want to take drug once every month? Ibra
band route
to close duct ? indomethacin iv
SE of amiodarone? peripheral edema
drugs can't take together? losartan enalapril
pt with blood pressure 175/90 what to do? ask the nurse to find an
urgent appointment to him
questions about different cases but all in community pharmacy all of

Ift them asking u to despine drug - don't! tell them to see the DR n
pt. with depression telling u that he want to die and .. blablabla what to
ooh

O
say? (1)OK its depression (2)through ur self in garbage?! (3)when did u
start feeling that way (4)no don't think god u r young
Celecoxib MOA selective inhibition of cyclooxygenase-2 (COX-2)

doxycycline and anapadline at what time better to avoid doxycycline


stomach SE

IFTect
type of cost for being in hotel while..? non direct non
nurse asking about TPN that's turn to be slightly collid ... was clear
Medical

wood
before what to do? discounte or continue or slow rate ..etc
70- How long does it take for the body to remove 99% of the drug by
first order elimination process after a single dose?
A) 2 half-lives
B) 4 half-lives
0
C) 7 half-lives
D) 10 half-lives
Studies:
Side effect of drug: randomized control
trials
Rare disease: case control
Risk factor: prospective cohort study

407-The Arabs were old used kohl for children What is the
harmful substance in kohl ?
A- lead
B- mercury
C- zinc

408- clostridium difficile cause?

A- salmonellosis

B- typhoid fever

C- pseudomembranous colitis

D- cholera

Note: treatment vancomycin or metronidazole

389- Which of the following marketing mix contains market


coven
assortments
A-Product
b- Place
O
C-price
D- Promotion

https://t.me/SLPESTUDY
388- Case of status epilepsy already got lorazepam IV. what drug
next?

O
A- phenytoin IV
B- diazepam orally
C- phenobarbital iv

297- Pseudomonas aeruginosa in gram stain ?


A- gram positive cocci
8- gram nigative cocci
C- gram positive bacilli
D- gram negative bacilli
O
Medication used with exercise induce asthma ?
A- Tiotripium
B- Cromlyn
C- Cortisone
D- salmeterol Salbutamol new
amoglobin analysis, positive IGG and IGM pollination, and
the type of infection ?
- A-An old infection
8- The condition worsened
C- New condition
D- The earliest new case does not appear with the new
infection A
281- When stop using Fingolimod for pregnancy ?
A- Before 2 month
B- B- 6 month Safe for Pregnancy
C- Before 6 months
case . pt DM 2symptoms .. went to lose weight..HTN ,heart
failure .. do life style change and excises drug choice for DM 2 ?
A- Insulin

O GLP I
B- liraglutide
C- metformin
C- Glypuride
agenist

https://t.me/SLPESTUDY
294- diagnosis of STEMI ?
OA-elevated ST segment with increased troponin D
B- elevated ST segment with increased bnp
D- elevated ST segment with increased CK

351- which the following use in central vine ?


A-D5w
B- 0.9 normal saline
C-3%normal saline
O
D-D10w
260- 2 year~ skin infection with penicillin allergy should take ?
A- Doxycycline
B- Amoxicillin + clavulanic acid

0D- Sulfamethoxazole + teimethoprim


bacteria
276-The best study design to determine side effects of drugs ?
A- case control
8- Cohort study
C- systemic review
0E- randomized control trial
RCT
182- What is the maximum beyond use date of rebackage drug
A-3 months
B-6months
B- 9 months

201- amount of water in neonate?


A-60%
8-75%
C-80%
D-90%
242- vaccination case he came for you to take mmr vaccine
history of taking immunoglobulin for hep a last month what to do ?
A- give immediately
8-tomorrow 3 months between
C- after 2 months
live vaccine
Ig and
C- after 6 months
D-
4- which enzyme inhibits bacterial cell wall ? lyzosome

148- Which of the allowing mood stabilizers would be most


appropriate in/ patient with liver disease?
0
A-lithium

https://t.me/SLPESTUDY
B- valproic acid
E- carbamazepine

target vancomycin trough in sever infection ?

5-10
B- 15-25
C-15-20
O
125- Which the following is needs a double check ?
A- Anti inflammatory
B- Anti depression /
C- Anti thrombosis
Antibiotics

111- OM patient on insulin but he is not taking his dose, his


HbA1c is
12%, he refuses taking insulin right dose be of weight gain what
do you do as a Pharmacist?
A- Refer to physician to decrease his insulin dose
8- decrease his doses
C- try to change his beliefs on insulin
F- tell him it's ok to not take his dose

110- Medication for osteoporosis in menopausal women


given
monthly?
A- Zoledronic acid
B- ibandronate
o
C-raloxifene

104- case about a girl has sore throat (mild) without any
infection
or other symptoms what should you advise her?
O
A- Gargle With Salt And Water ~
8- Clindamycin
C- Anti-Histamine

https://t.me/SLPESTUDY
D-Ceftriaxone l.v Every 12 Hrs
86- what is the Clozapine test monitoring ?
O
A - Neutropenia (ANC) ~
8 - Liver Functions
C- Kidney Functions
D-Testing Of Vitamin 812 Levels

84- Which of the following Drugs is used for positive symptoms of


schizophrenia ?
O
A- Haloperidol
B- Levodopa
C- Sertraline
D - Citalopram SH
For ve sypto
y
Tanner

72- Breast cancer.the gene HER2 was negative we can use?

0
A- Letrozole
B- -Trustuzumb

70 -patient on enternal nutrition came with lab tests, low


phosphate low magnesium ?
A- Refeeding syndrome t Pou ang
O
67- patient has severe endocarditis with staphylococcus aureus
on vancomycin but sensitivity test is done and the patient is only
sensitive on linezolid but it's not FDA approved in endocarditis
what's the management?
Increase vancomycin dose, ~ 't give linezolid, give
A- linezolid as off-label
O
https://t.me/SLPESTUDY
63- epimor of glucose ?
A- Ribose
b- galactose
O B- deoxyribose
50- Type of hypersensitivity for drug induced hemolytic anemia ?

D
A- Cytotoxic reaction
type 11
b- immediate sensitivity reaction

A. labetolol
nonselective
O
B. terazosin
C. clonidine 9
agonist
D. captopril Acf I
96- A 6-year-old boy is brought with complaint of uncontrollable spasm-
like movements and excessive eye blinking for four months. Chronic
motor disorder was diagnosed.Which of the following is the best
management?
A. pregabalin
B.haloperidol
C.methadone
0
D. chlorzoxazone muscle relaxant Relaxon
13-The synthesis of glycogen from glucose called?
-_A- Glycogenesis
50- A 62-year-old patient, otherwise healthy, was brought to the
Emergency Department in a confused state after ingesting one green
fruit of castor bean two hours ago. He had vomit once at home. He had
dryness of mouth and the pupils were bilaterally dilated with sluggish
reaction to light. He was treated symptomatically with activated
charcoal. Which of the following is the main active constituent
responsible for the toxic effect of castor seeds?
A. Ricin
B. Barbaloin
C. Mezerein Castor _Ricin
https://t.me/SLPESTUDY
D. Ergotamine
3- Which of the following is the most likely mechanism of carbon
monoxide poisoning?
A. inhibits the gag reflex
B. paralyzes the muscles of the diaphragm
C.decreases the oxygen-carrying capacity of the blood
D. reacts withh amino acids in the body to form ammonia
29- Which of the following is the main purpose of levigating a
suspension during extemporaneous compounding?
A. improve flowability
B. reduce particte size
C. reduce the zeta potential
D. prevent microbial growth
76- Which of the following drugs would decrease the amount of
minimum alveolar concentration (MAC) for an anesthetic drug?
O
A. diazepam
B. buspirone
C. ephedrine
D. loratadine diatepam Mac
46- What dose linear pharmacokinetics mean?
A. The clearance of the drug is not changing when dose increases
B. The plasms concentration versus time curve is a straight line
C. The half-life of the drug is equal to its volume of distribution

time

https://t.me/SLPESTUDY
D. The drug levels in the body are in steady state

340 0
F AUCpoX Noo 2 100
Absolute 20 4
AVCN X DOK PO
61 100 85
80
86-Carbamazepine (pKa=13.2) is an antiepileptic drug used in the
treatment of status epilepticus. This condition requires fast intervention
with carbamazepine injection.
Which of the following is most likely to describe what will happen if
carbamazepine was given orally?
A. Fast intervention because the drug will be ionized in the stomach

O
B. Slow intervention because the drug will be ionized in the stomach
C. Fast intervention because the drug will be unionized in the stomach
D. Slow intervention because the drug will be unionized in the stomach
79- Which of the following drugs complexs with dairy products or with
antacids when administered orally?
A. Enalapril
B. Warfarin
0
C. Doxycyline Tetracycline
D.Acetysalicylic acid
71-Which of the following is the term used to describe the
pharmaceutical technique of grinding an insoluble substance to fine
powder while wet ?
A. Trituration
O
B. Levigation
C. Attrition
D. Milling

https://t.me/SLPESTUDY
31-Which of the following is used for the treatment of neo-vascular age
related mascular degeneration?

o
A. Aflibercept
Vascular
B. Latanoprost
Neo
C. Muromonab
D. Gemtuzumab
A Fli
4-Which of the following conditions is a serious adverse reaction of
fluoroquinolone therapy?
A. Eczema
B. Bleeding
C. Depression

e
D. Q-T interval prolongation
93- A 68-year-old woman with type-2 diabetes, neuropathy and seizures
has been newly diagnosed with depressive illness. Which of the
following medication would be best to initiate?
A. Bupropion
B. Duloxetine SN
O RI
C. Paroxetine
D. Amitriptyiine
85- Which of the following vaccines is specifically indicated for all
pregnant woman during each pregnancy, in addition to influenza
vaccine?
A. Hepatitis B vaccine
B. Hepatitis A vaccine
C. Pneumococcal vaccines
O
D. Tetanus, diphtheria, pertussis
Tdap
86- A 55-year-old man comes to the pharmacy complaining that has
acetaminophen with codeine he started 2 days ago to manage has post-
operative pain does not seem any better than when he uses
acetaminophen alone. After the pharmacist reviews his profile, he
noticed results from pharmacogenomics testing performed 3 years ago
that shows he is a CYP2D6 poor metabolizer.
Which is the best explanation why this patient does not seem to benefit
from codeine?
A. Codeine excretion is rapid
x
B. Codeine absorption is slower r
o
xC. Codeine is metabolized faster
D. Codeine dose should be increased
a
https://t.me/SLPESTUDY
81- A 65-year-old.diabetic man admitted to Emergency Department (ED) with a 3-day
history of productive cough, shortness of breath (SOB) and chest pain. Chest X-ray
showed bilateral lower infiltration
Which conformed the diagnosis of pneumonia. An H1N1 virus was detected in nasal
swab.
Which of the following antivirals is the drug of choice in this case?
A. Ribavirin
B. Acyclovir
C. Oseltamivir
D. Gancyclovir

67- Which of the following vaccines is used to prevent cervical cancer ?


A. Zoster
B. Varicella
C. Human papilloma-virus
D. Tetanus, diphtheria, pertussis

O 0
O BMI 8 31.2

https://t.me/SLPESTUDY
(47) Which of the following could lead to decrease in international
normalization ratio (INR) in a patient of warfarin therapy?
A.decrease the consumption of dietary vitamin K

OB. carbamazepine therapy V


C. cotrimoxazole therapy CYP inducer
D. metronidazole therapy
(38) A 33-year-old woman with no underlying medical condition came to
a clinic in with complaints of dysuria, urinary frequency and urgency, and
suprapubic tenderness. She was diagnosed with urinary tract infection
and treated with trimethoprim/sulfamethoxazole. Which of the
following is an objective parameter to assess the effectiveness of drug
therapy?
A. resolution of back pain
B. decrease in cough frequency
C. improved ability to concentrate
D. normalization of white blood cells
028-which of the following is considered first line therapy for the
management of partial seizures?
O
A. Carbamazepine
B. Phenobarbital
C. Gabapentin
D. Primidone
14-A 65-year-old man was diagnosed with type 2 diabetes five year ago,
maintained on metformin
Which of the following need to be monitored?
A.Vitamin c level
B. Vitamin D level
C. Vitamin B1 level

O18-what is the recommended prophylactic therapy for pregnant women


D. Vitamin B12 level

colonized with group B streptococcus in labor with penicillin allergies at


high risk for anaphylaxis?
A. Cefazoline
B. Linezolid
C. Penicillin G
D. Clindamycin r
O
https://t.me/SLPESTUDY
6-An 18-year-old boy with a history of illicit drug use is suspected to
have an overdose of benzodiazepines and is experiencing ataxia,
drowsiness and nystagmus .
Which of the following would be the best antidote?
A. Activated charcoal 50grams
O
B. Flumazenil 300 mcg intravenously
C. Naloxone 400 mcg intramuscularly
D. Haloperidol 10 mg intramuscularly

70) To which of the following the major histocompatibility complex


unique?
A) Each cell
MHC
B) Each organ
C)Each species
D)Each individual
38) What is the type of costs for traveling from Jeddah to Riyadh to
health care from pharmacoeconomic perspective?
A) Indirect
B) Intangible
C) Direct -non medical
D) Non-direct medical
1-A drug has a volume of distribution of 35L in a 70 Kg man.
Which of the following best describes Its distribution?
A. it is bound to DNA
c
B. it is dissolved in lipids
C. it has low bloavailability
D. it is mostly distributed in plasma

https://t.me/SLPESTUDY
24- What is the main absorptive function of the colon?
A. Iron
B) Triglycerides
C) Water and glucose

0
D)Sodium, chloride, and water NaCl H2O
(8)What is the estimated amount of plasma in a 70-Kg male? (50-
55ml/kg)
55 70 3850 ml 1000
0
A)-3.5 liter

85L
B)-1.1 liter
C)-35 liter 3
D)-11 liter
(13)Which of the following parenteral anticoagulants require routine
monitoring of coagulation lab parameters?
A Ptt
0
A) UFH intravenously
B)UFH subcutaneously
C)enoxaparin subcutaneously
D)fondaparinux subcutaneously
5)Which of the following vaccines is recommended for people at the age

A) hepatitis B vaccine
0
B)pneumococcal vaccine
C) meningococcal vaccine
Ppsv 23
D) DTaP vaccine (diphtheria,tetanus,pertussis)
1)Which of the following Is the effect of smoking on serum olanzapine
levels?
A)has no effect
B) Increase in serum olanzapine levels
a
C) decrease in serum olanzapine levels
D)decrease In olanzaplne levels initially then increase in It
100)Which of the following is a pharmacological property of
chlorpromazine?
A) antihistamine r
B) anti emetic drug
C) cholinesterase inhibitor
D) selective serotonin reuptake inhibitor

https://t.me/SLPESTUDY
82-which of the following is the best classification for Bisoprolol?
A) beta 1 adrenergic receptor agonist
B)beta 1 adrenergic receptor blocker
C)alpha 1 adrenergic receptor blocker
D) non-selective beta adrenergic receptor blocker
60-What is the Latin abbreviation for "after meals"?
A) A.C.
B) A.A.
C)P.C.
D) I.C.

85- In pharmacoeconomic studies, the costs (inputs) for types of


analyses are measured in dollars but the pharmacoeconomic method
selected will depend on how the outcomes are measured.
Which of the following analysis described the method when patient
preferences for alternative treatments are being considered as the
outcome?
A) Cost-Minimization
B) Cost-Effectiveness
C) Cost-Benefit
D) Cost-Utility
56) A 24-year-old male presented to the hospital with a skin infection
the microbiology technician received a sample form this patient the
sample in a special media was incubated over 24 hour and showed
Staphylococcus aureus. The microbiologist used disk diffusion method to
determine the susceptibility of different antibiotics (see image)
What is the most sensitive antibiotic?
A)Ampicillin(Amp)
B)Vancomycin(Van)
C)Methicillin(Meth)

https://t.me/SLPESTUDY
D)Penicillin(Control)
52) Which of the following inhibits gluconeogenesis?

O
A)Insulin
B)Glucagon
C)Epinephrine
D)Glucocorticodes
31) Which one of the following medication can a gynecologist prescribe
to a patient ?
A)Tamoxifen
B)Fingolimod
C)Anastrozole
4 6
O
D)Misoprostol
19) Who is responsible to promote the health in Saudi Arabia?
A)Saudi Food and Drug Authority (SFDA)
B)Ministry of Education (MOE) no
O
C)Ministry of Health (MOH)
D)Health Colleges
3) Which of following studies are more susceptible to recall bias?
O
A)Retrospective cohort
B) Prospective cohort study
C)Randomized controlled trial
D)Non-randomized controlled trial

Table spoon
- Alkaloids digitalis
Phase
Oral birth control
? Chlorpromazine side effect
Dry mouth
-

https://t.me/SLPESTUDY
1200
Cost analysis

CHA2DS2-VASC
- Crcl |
- Spicefic gravity I
Biopharmaceutics classification
- Permeability and Solublity
High Low

O
Hit L H

H L LL

Which anti-psychotic patient has involuntary movements, gain weight,


can't sleep, isolated from people must take?

C
Clozapine then Haloperidol
3
- Cause Agranulocytosis? clozapine

https://t.me/SLPESTUDY
- in large amount of parental medication, we use? Sterile water for
Injection
- otitis media in children? High dose of Amoxicillin or Azithromycin .
- Drug make complex with dietary products? ciprofloxacin
- Drug make complex with antacids? tetracycline and Fluoroquinolones
and Doxycycline.
- Pt takes ciprofloxacin, you advised him to ? (administer one hour
before or two hours after phosphate administer)

my
- Aspirin dose with Ticagrel ? 325MG

- Beta blocker with intrinsic sympathomimetic activity? pindolol

O
- Syncope may occur with first dose of? Prazosin
- Drug causes water retention ?
Indomethacin
- Isotretinoin contradicated in ?
pregnancy
- Your advice for patient takes Alendronate?
take it 1/2 hr before breakfast with water and star upright for 1/2 hr
is
2
- The best drug delivery in infant? Neutralizer
- Another name for Vit C ?

I
ascorbic acid
- Patient with BPH Benign prostatic hyperplasia ? Tamsulosin
- Tolerance ? increase dose of drug to obtain the same effect
- Viral vaccine?
Inactive and attenuated
- When Vd increased ? Drug is high in tissues
- Indirect adherence ? pill count

https://t.me/SLPESTUDY
or latanoprost
betaxolol
- Glaucoma with asthma? Brimontidine or lonatprost
me
- Phase 3 ? Repolrization due to the k outflux 6 461
L
ttt
- T helper? CD4
is brimonidine
- Less important for breastfeeding? MycBA
syndrome
Reye's
- Treatment of rhynus syndrome? CCB
- Leafy Green Contradicated ? in Warfrain
take it in fixed
- Vaccine can be frozen? Varcilla amount not CI
- Aspirin in Bathroom? hydrolysis
- ttt of jetlag? Melatonin
- Phenobarbital duration action? Ultra short
long actingT
- Menopausal woman? Monthly.. Ibadronate Yearly.. Zalonderic acid,
- - Treat neuropathic pain and depression?
Duloxetine
SNRI
- Uterine stimulates? ergot alkaloid

E
- Pregnant with UTI + GOPD? Cefuroxime

https://t.me/SLPESTUDY
13 April 2022 -

- Target hemoglobin in CKD on Epotin? ( g/L)  10-12 g/dL 11g IDLE


-
- Case E.coli with CKD? Ciprofloxacin 400 mg IV Q 24 h
than
Antidepressant effect? 2-12 weeks the correct answer 4-6 weeks
To 12weeks
- Pain scale in children? Faces pain scale
- Recently published studies? PubMed uptodate pubmed V
- Amphotricin undergo? hydrolysis

2305
- Antibiotic inhibit protein synthesis by 30 what effect? Initiation
- Clozapine monitoring test? Neutropenia ANC
- Drug CI in crcl < 20 ? spironolactone
- Which of the following antiplatelet working by inhibition of Adenosine
Diphosphate Receptor? Clopidogrel
ADP inhibitor
- case about a girl has sore throat (mild) without any infection or other symptoms
what should you advis? Gargle With Salt And Water
- oleaginous base for ointment? White petroleum
i
-
-
woppy
Which of the following to be used with caution in renal failure? Sitagliptin
Hand shacking= physical communication
- Most source of infection in hospital? Healthcare practioners hands
people
- Why ephedrine prefereed over pseudophedrine?
- Antiepleptic in pregnancy lamotrigine or levetiracetam
- Mission and vision in which strategy? Strategic planning
- boxed warning when using for teeth pain in infant lead to seizure and death ?
clove oil
- Which contraceptive have high dose of estrogen and contraindication in DVT

-
womeo ? Transdermal patch
o
Head of pharmacy decide to make group to arrange formulary drugs in hospital?

O
planning
13 April 2022 -

- which of the following can assest in regulation of sleeping pattern? doing aerobic
exercises for 10 mins
- a l
Case about patient withfrunnynose\and complain about and ~ need fast relief of
congestion ?

Tm
- Pt wit metabolic alkalosis and edema use? acetazolamide
Alk
- Parkinson patient has dfY cough he was using Selegline
- MAOI but stopped it before{§).veeks what to do? Dextromethorphan
- What is required in a narcotic prescription options? single narcotic agent written
at the top with red pen

AcEnal
_I
- acetyl-CoA to form malonyl- coA which vitamine is used? biotin (B7)
- To reduce bias use? Anonymous questionnaire
- Oral furosemide to IV
Wipo i2
- Ginger for nause in pregnant
- Heart rate goal in Atria fibrillation pts taking
60 to 80 Aim for HRC110bpm
- Statin interact with greapfruit SimbaSta
- P450 Inhibtor? Cimetidine

- Rasagiline contraindicsted in hepatic pts

2 3
- Warfarin gene ? CYP2C9*2 ,,CYP2C9*3 2cg 2cg
- Clopifpgrel gene ? CYP2C19

- Pharmacovigilance definition is defined as the science and


activities relating to the detection,
assessment, understanding and
- RCT prevention of adverse effects or
any other drug-related problem.
CCB
- Antidote for: iron, beta blocker, BDZs, Flumazenil

Glucagon
Deferoxamine
13 April 2022 -

- Drug cross placenta? Amoxicillin

- Ipeac for
A bias is

- First pass metabolism? Oral


o
- Celecoxib contraindicated in CVD

I
- Ca dose in geriatric 1200 mg

Cha2ds2vasc =atrial fibrillation 1200mg


- Score for stroke 2 point

- Clozapine ANC
monitor ANC
fibrinolytic
- Used in Acute coronary syndrome? Alteplaee
I in NSTEMIK
- Pt taking warfarin what is best advice regarding green leaf? Maintain same

HiT
amount

- Belladonna atropine anti

- Which to give in type 4 hypersensitivity ? Histamine


00A Epinephrine
-
Rosuraf
Statin that interacts with grapefruit: rosuvastatin, fluvastatin, pitasratin

- ISMP for medication error Safe with


r
grapefruit
Ismp ME
13 April 2022 -

Contact Dermatitis
type IV

for visually inspection

HCQ
- Hydroxychloronr ocular monitoring

- Vincristine label ( for IV only fatal interathecally)

- Warfarin: type 2 metabolism

- Activator? Platelet activating factor

annualfecaloccultbloodtesting
- Colorectal cancer screening at age 50? Recall ocult blood test
starting at age45

MCU
- b7between methycoA to
Biotin

- Max dose of ergotamine in the attack 6 MG


I 6mg 3 tablets

- Hormone produced by posterior pituitary gland


ADH Oxytocin
- Sencondary lyphotic organ? Tonsils, spleen, bone marrow, thymus gland

- Best description of medication adherence?


13 April 2022 -

I
- Well controlled blood pressure in which part of SOAP? Assessment

- Next appointment in which part of SOAP? Plan

HLA B 5801
- Lamotrigine : 5801 = cutaneous
I
g p alone
- Best contraceptive for women with history of DVT? Levonorgestrel

-
In
Interaction between aspirin and Amytrip time
9bleeding
- Drug induce ulcer? Indomethcin
NSAIDs

- Drug cause fatal abnormally in pregnancy ? finasteride

- Best time to take insulin glargine? with evening meal


or at bedtime
em
- 30s? Initiation

-
I
Statin side effect? Myopathy
a a.m
Cpoiten
- Epotib target in g/L?? less than H 91dL in Hg
- Drug for supraventricular?? verapamil and diltiazem

CCB w Adenosine
or PB
13 April 2022 -

IV fluids consider WBI


- Spiroloancto -> hyperkalemia Wine
- Lithium toxicity? Hemodialysis Hemodialysis if
severe toxicity seizure Cona
- Laxative for chronic constipation? Renal
insufficiency

- Pregnant with otitis media? Azithromycin

Q WH
- Levothyroxine counseling was
at least 60 minutes before breakfast

- Antihypertensive safe in pregnancy


methyldopa, labetalol, and nifedipine XR hydrahazine
- Why aspirin should be avoided i

- Pethidine = psychotropic drug

- Hashimoto symptoms?
same as
hypothyroidism
Vala prodrug of Acyclovir
- Valacyclovir is a pro-drug for Ba
valacyclovir, meaning that it is turned into
acyclovir in the body

Na HCOs
- Antidote for lithium? Sodium bicarbonate

IV fluids
13 April 2022 -

- Antiprogesterone drug? Mifepristone


Antifungal
-
F
Pregnant with itchy vulva? Cephalecixin maybeClotrimazole
- Antiaythmic drug category 3 MOA ?K-channel blockers
KCB
- ST means? interval between ventricular depolarization and repolarization
se

Is µ
affentity
quinine

-
I
To asses severity of HIV? CD4 cells

-
I
Drug that interferes with the normal function of microtubule growth? Paclitaxel
a

- First line antiemetic in cancer patient? ondansetron

- True solution also known as? Homogenous

Bactrian Sutfouglureafurosemide
- Drug contraindicated in sulfa allergy
- Rifampicin change urine color Red color urine
-
w
Antidepressant for obese girl? Bupropion
- Phenytoin is dose-dependent Zero order kinetic
- What is faculative halophiles? an organism that that survive and reproduce at high salt
concentration, but also can survive and reproduce at not-high
- Pregabalin halo _Salt salt concentrations.

- Antidibetic need renal adjustment? Sitaglipton


DPP ai
- Cushing syndrome: low adrenal
Cortisol
- Hashimotos symptoms: bydrycardia
bradycardia
T
13 April 2022 -

- Virus genom: DNA or RNA


- Osteomyelitis pathogenic causative: E. Coli StaphAureus pseudomonas V
- domestic pigeons: Chlamydophila psittaci
- Baby with Rota virus? ORS
- Strategic
- Organizational
- Oh in type III
- Ig in milk:
As MiG
- Warfarin elimination by liver
Cyp 2cg
- Disadvantage of primary literature? Require good skills
- DASH diet: HTN
- Medication cause orthodontic hypotension? Nitroglycerins
- Another name of vit K? Phytonadione 
- Lupus erythrmerasos: autoimmune disease
s's
- Antidepressant 2-12weekes to see results
b Ieee Us
-
w
Causality score: narnjo
- Accuracy definition
- Mortality = death rate
- Provoked = 3 months
DVT
- Metformine should be stopped before contrast media use due
- Interaction CCB and calcium too si
- Trough level in severe cases: 15-20 Vancomycin
-
J
Trough level drawn for safety
peak efficacy
- Price, promotion, product, place

up's
13 April 2022 -

- Handshaking= physical communication


- Titration:
- Levigation: grin
ding
5
- Decoction: extraction active material from plant by boiling
- Reconstitution:
- High alert medication:
- Vacccines before Haij Meningitis 
no
j
Model of healthcare in Saudi? Free
- Fluid for ringer lactate
- Emulsion preparation: Wet gum method, dry gum method, continental
- Oleaginous base? White petroleum

mono fit want

- Structures:
- Pystigmaun ..
- Calculations:
- CrCl
- Tablet number
00
- Infusion rate
- Drops
- Absolute bioavailability
- BMI
- Strucyre methyphenidate
- Risk factor DVT: surgery
EE
EMmethylphenidate
EStructure
w
13 April 2022 -

-
-
O
Drug used for both bone and GIT problems? Ca carbonate
Drug banned in sport competitiors by international Olympics? Furosemide
- Levofloxacillin coverage: gram positive and negative ve
ve
- Budget statement
- Model healthcare Saudi? Free
- Newly medical rep? Can not access any healthcare institution (limited access)
- Peripheral neuropathy in diabetic pt? Monofilament testing
- Supplement for pt with colitis ? Calcium and vit D
Triage definition
Absolute contraindication
cardiac arrest
- Accuracy definition
- First line antiemetic in nausea pts
- Benzodiazepines oral absorption?
- Peak= efficacy
- Trough= Safety

-
13 April 2022 -

- Conversion IV furosemide to oral (PO:IV 2:1)

- Which of the following systems can be integrated with computerized physician


order entry to guide physician during prescribing medications?
Clinical decision support system

- Which of the following method is successful in reducing medication error and

T
enhance patient safety in hospitals?
Implementation of computerized physician order entry (CPOE) system

- Which of the following pharmacoeconomic studies is the simplest because the


outcomes are assumed to be equivalent and only the costs of intervention are
compared?
Cost-minimization analysis
CMA
- Which of the following vaccines is contraindicated live attenuated?
Measles ,Mumps ,Rubella (MMR)

- Which of the following study design is the best to use to assess the awareness on
the usage of vitamin D supplements among people of Jeddah?
Cross sectiona study
13 April 2022 -

- Which of the following method is successful in reducing medication error and


enhance patient safety in hospitals?
Implementation of computerized physician order entry (CPOE) system

- Which of the following enzymes is responsible for forming DNA copy from RNA?
Reverse transcriptase
RT RNA
DNA
- Which of the following is the benefit from adding a conjugate of polyethylene
glycol (PEG) in some medications like Peg-interferon?
To extend the half-life of medications
9th
- What is the Latin abbreviation for "after meals"?

o
P.c

209 1000 20,0009


- Penicillin-G potassium Is available in 50 ml vial containing 20 gram. Each
my of penicillin-G potasslum is equal to 1500 units. How many units of
milligram
penicillin G potassium are there in each vial?
Lng 1500units
30,000,000 Units 20g000mg X units
30,000,000units
-
2
A 4-week Infant is diagnosed with patent ductus arteriosis. The doctor wants to
avoid performing surgery. Which drug will be administered to avoid surgical
closure? Indomethacin

- Which of the following is the most appropriate time to draw a plasma blood
sample for digoxin monitoring?

6
6 hours post dose

dig hrs post


13 April 2022 -

- A 62-year-old man who just had a needle biopsy of pancreas adenocarcinoma. A


pharmacist saw his brother in the hall, and him not to tell the patient because the
knowledge would be very on his health.A family conference to discuss the
prognosis is scheduled for later that afternoon. What is the way for the
healthcare practitioner to handle the situation?
The healthcare practitioner should honor the request of the member who is
protecting his beloved brother from the patient

- A 38-year-old man is using citalopram 20mg po daily for depression which he has
been taking for 1 year. A week ago, he had an infection and was prescribed
linezolid 600 mg PO q12h for a total of 14 days. The patient began to get
confused and easily agitated like he was breathing rapidly and started to sweat
and developed. He was noticeably shaking. Which of the following is name of this
drug-drug interact happened to this patient?
Serotonin syndrome
SSRI MAOI Serotonin syndrome
- A 74-year-old man with chest infection is admitted to a teaching.The patient
requires chest an X-ray. The chief resident bronchoscopy to be conducted. One of
her reasons for this decision allow an inexperienced intern to practice this
technical procedure.Which of the following describes his action?
The senior physician should remain present when inexpen students make this
procedure

- A pharmacist working in inpatient pharmacy received a request pharmacy and


therapeutic (P&T) committee to evaluate a included in e hospital formulary. The
request wants to have the value in a monetary value(dollar value) Which of the
following pharmacoeconomics tools will the pharmacy to answer the request?

CBA
13 April 2022 -

Cost-benefit analysis

- Which of the following is the appropriate defined pharmacoepidemiology?


The study of the use of and effects of drugs in large number people.

- Which of the following is a common enzyme polymorphism that could potentially


alter clopidogrel efficacy?
CYP2c19

- Which of the following drugs can be safely recommended to a patient that has
been suffering from chronic constipation?
Ispaghula husk

- A 45-year-old male is admitted to the hospital for craniotomy Phenytoin has


been prescribed to him after the surgery. Before initiation of the drug the
pharmacist suggested to do pharmacogenetics test and the results revealed that
the patient carries the HLA-B*58:01 allele. Which of the following adverse effects
is associated with this patient's allele type?
Severe cutaneous reaction

- Which of the following is considered as live attenuated vaccine?


Measles, mumps, rubella. (MMR)

- A 30-year-old woman with a history of deep vein thrombosis that was treated a
year ago wants to use birth control pill. Which of the following contraceptive pills
would best suit her?
Levonorgestrel
13 April 2022 -

- A 60-year-old man with 6-year history of myocardial infarction and gout. He


presented to the clinic complain of severe pain in his first left consistent with
acute gout flare. He mentions that he can't colchicine. What is the most
appropriate pharmacotherapy recommendation for his acute flare-associated
pain?
Former
A. Ibuprofen. B. Allopurinol. C. Indomethacin D. Acetaminophen

- Which of the following vaccines is used to prevent cervical cancer ?


Human papilloma-virus
HPV
- Which of the following drugs is contraindicated in heart failure?
Celecoxib
- An 18-year-old obese girl with newly diagnosed depressive illness is being advised
to start antidepressant treatment. Which of the following is best to initiate?
Bupropion

- Which of the following genetic testing might derive a potential benefit from
transtuzumab therapy in a breast cancer patient?
HER2 human epidermal growth factor receptor 2. /neu genotype positive

HER ve
13 April 2022 -

- A 50-year-old Saudi woman with hypertension and dyslipidemia. She presented


today to her follow up visit with family medicine clinic. Her sister has a type 1
diabetes mellitus. No alcohol or ciggarrate use is reported. She is physically active
for 2 minutes a week. (Wt: 51 kg, ht: 160 cm)??
Family history

- Thalidomide is apotent immunomodulatory drug used to treat leprosy


Thalidomide exist in two conformation structures with different pharmacological
and toxicological activates teratogenicity is believed to be caused on
conformational structure (see image ). Which of the following is the most
appropriate term used to call the two conformation of the thalidomide?

OR
Enatntimomers
S
- What is the age range of a neonate?
Birth to one month
- Which of the following is a potassium-sparing diuretic (Hyperkalemia)??
Spironolactone
- Which of the following anti-cancer alkaloids act by stabilizing the microtubule
polymer and protecting it from disassembly?

-
O
Paclitaxel
Which of the following is the term used to describe the pharmaceutical technique
of grinding an insoluble substance to fine powder while wet?

-
a
Levigation
Which of the following type of water should be used for reconstitution of
parenteral preparations?
Sterile water for injection
Swift
13 April 2022 -

- Which of the following solutions is required for the reconstitution of parenteral


formulations?
USP standard sterile water for injection Swift
- Special techniques are required during compounding of emulsion. Which of the
following techniques is used for compounding of emulsion?

O
Dry gum

- Which of the following is another name for a true solution?


homogeneous mixture

- Which of the following is alternative name for vitamin B6?


pyridoxine

- A 12-year-old boy is brought to the Emergency with complaints of inability to


open eyes and difficulty in breathing. History revealed snake bite. Examination
shows anxiouus, tachycardia and tachypnea, bilateral ptosis, no bite mark seen
and no swelling of the.limb (see lab result). Clotting time: 20 min ??
Give Anti-snake venom (ASV) plus Neostigmine and keep under observation

- Which of the following is a recommendation to reduce the inadvertent


intrathecal administration of vincristine?
Dispense vincristine in mini bag instead of syringe

- Which of the following is the main purpose of levigating a suspension during


extemporaneous compounding?
d particle size
13 April 2022 -

Reduce particte size


- Which routes of administration result in faster absorption?

-
O
Rectal
Which of the following drugs exhibits dose-dependent pharmacokinetics at
normal therapeutic doses?

T
Phenytoin zero order kinetics
- Structure of ciprofloxacin (see Image).Which of the following is the benefit of the
piperazine group at positian??
Improves the spectrum of antibacterial activity (especially Gram-negative) r
- Cleaning is a very important step when working with laminar flow hoods.Which
of the following disinfectants is most suitable for cleaning the hood before
compounding a sterile preparation?
70% isopropanol

- Which of the following medications is an anti-progestin?


Mifepristone
- Which of the following statements describes unfractionated heparin?
mucopolysaccharide polymers
- Which of the following vitamin deficiency can cause scurvy?
Vitamin C

- Which of the following pharmaceutical activities should follow the USP chapter
797 standards?
Compounding of sterile preparations
13 April 2022 -

Nilo phila
- What is the therapeutic indication for nilotinib?
Philadelphia chromosome positive chronic myelogenous leukaemia
CML

- A patient was admitted to the hospital due to poisoning symptoms after ingesting
an unknown herbal remedy from unknown source. The herbal drug has been
submitted to the analytical lab for analysis. The laboratory phytochemical
screening shows the following results: Baljet test +++++ ??

-
o
Digitalis
In case a patient develops toxicity from midazolam. Which of the following
medications can be given as an antidote?
Flumazenil

- D-Galactose and D-Mannose are monosaccharide sugar. They have the same
number of carbons and the same functional group, but they differ in their
configurations. This reflects their pharmacoceutical uses (see image) Which of
the following types of stereo-isomerization do these sugars represent?

0
Diasteromers

- A water soluble vitamin catalyzes the carboxylation of acetyl-COA to form


malonyl-CoA, which is required for the synthesis of fatty acids (see image) Which
of the following vitamins plays this role?

0 Mal
Biotin (b7)
A c

- Which of the following is the exact date for a drug to be safety taken if its
expiration date December 2018?
December 31, 2018
13 April 2022 -

Aldosteronn renal tubular


Thalidoamide ensn enantiomer
Law thalido
Levigation: grinding an insoluble substance to fine powder (dec. size)
Heb A vaccine: 3 Doses
Another name of vit b6: pyridoxine
Lithium to its antidote: sodium bicarbonate

o to

MINE
I MiPRA

You might also like